Diese Woche gab es ja den Nobelpreis für Zeilinger, Clauser und Aspect für Forschungen zur Quantenverschränkung. Und natürlich überschlagen sich wieder alle darin, zu erklären, wie unverständlich diese Quantenverschränkung doch ist. (Florian Aigners Erklärung ist eine löbliche Ausnahme und vermutlich so ziemlich das beste, was man in 2 Minuten im Fernsehen umsetzen kann.) Grund genug, hier mal ein wenig zu erklären, was es damit auf sich hat. Zur Warnung sage ich gleich dazu: “Unverständlich” im Sinne von “geht gegen jede Intuition” ist die Sache, daran lässt sich nicht drehen. Aber was das Seltsame an der Verschränkung ist, das lässt sich schon verstehen.

Ich erkläre die Sache wie so oft an einem Spielzeugmodell (das allerdings eine gute Näherung für eine tatsächliche Eigenschaft von Elementarteilchen ist, nämlich den Spin). Unsere Teilchen (also die Dinger, die sich nach den Regeln der Quantenmechanik verhalten sollen und mit denen wir die Verschränkung bauen) haben eine Art eingebaute “Richtung”, die man durch einen Pfeil kennzeichnen kann. Der kann in beliebige Richtungen zeigen, nach oben, unten, rechts oder links, schräg rechts oben oder so, aber nur in zwei Dimensionen. Der Pfeil ist also wie ein Zeiger auf einem Kompass mit Richtungen Norden, Nordosten usw. Für’s erste brauchen wir aber nur vier mögliche Richtungen: Oben, unten, rechts links (oder Norden, Süden, Osten, Westen, wenn ihr das lieber mögt).

Nehmen wir an, wir haben ein Teilchen, dessen Pfeilwert wir nicht kennen. Wir können die Richtung nicht einfach messen, das erlauben die Spielregeln der Quantenmechanik nicht. Wir können nur entlang einer bestimmten Richtung messen, beispielsweise in der Vertikalen. Wir können uns zum Beispiel vorstellen, wie haben einen Apparat, der alle Teilchen mit Pfeil nach oben nach oben schickt und alle mit Pfeil nach unten nach unten.

Unser Teilchen mit unbekanntem Pfeil trifft also auf den Apparat und wird dann nach oben oder unten abgelenkt. Schicken wir es danach auf noch einen zweiten identischen Apparat, wird ein Teilchen, das beim ersten nach oben ging, auch beim zweiten nach oben gehen, eins, das beim ersten nach unten geht, auch beim zweiten und so weiter. Daraus können wir schließen, dass ein Teilchen, das hinter dem Apparat nach oben läuft auch tatsächlich den Pfeilwert “oben” hat.

Das gibt Regel 1: Ein Teilchen, dessen Pfeilwert wir gemessen haben, hat auch tatsächlich diesen Wert, denn wir messen ihn danach immer wieder. Teilchen können also einen eindeutigen Pfeilwert besitzen.

Jetzt nehmen wir einen zweiten Apparat hinzu, diesmal einen, der die Zustände Pfeil rechts und Pfeil links unterscheiden kann. Wir schicken wieder ein unbekanntes Teilchen auf den Apparat, hinterher haben wir dann ein Teilchen mit Pfeilwert rechts oder links.

Interessanter wird es, wenn wir ein Teilchen mit Pfeilwert oben (oder unten) auf den Apparat schicken, der rechts oder links unterscheiden kann. Dann bekommen wir nämlich zufällig einen Wert und zwar mit derselben Wahrscheinlichkeit von 50%. Umgekehrt genauso: Schicken wir ein Teilchen mit Zustand rechts (oder links) auf den Apparat, der oben und unten unterscheiden kann, bekommen wir mit 50% Wahrscheinlichkeit oben und mit 50% unten.

Ein Teilchen mit Pfeil oben (oder unten) hat also keinen eindeutigen Wert für die Eigenschaft rechts/links und umgekehrt genauso, misst man die Eigenschaft, ist das Ergebnis zufällig. Man sagt auch, der Zustand “oben” ist bezüglich der Eigenschaft links/rechts in einer Überlagerung. (Mehr über Überlagerungen findet ihr hier.)

Regel 2 Die Zustände oben/unten und links/rechts sind nicht miteinander vereinbar, kennt man den Wert in der einen Richtung, ist der Wert in der Richtung senkrecht dazu schlicht unbestimmt. Ein Teilchen kann also bezüglich einer bestimmten Eigenschaft keinen eindeutigen Wert haben.

So, und jetzt kommen wir such schon zur Verschränkung. Wir denken uns eine Maschine, die immer zwei Teilchen gleichzeitig aussendet, am einfachsten in entgegengesetzte Richtungen. Wenn wir die Pfeilrichtungen dieser Teilchen messen, stellen wir fest, dass sie immer genau entgegengesetzt sind, ist das eine oben, ist das andere unten, ist eins links ist das andere rechts. (Technisch lässt sich das z.B. umsetzen, indem man Elektron-Positron-Paare erzeugt, das Beispiel nimmt Feynman in den Feynman Lectures.)

Nehmen wir an, eins der Teilchen fliegt nach A, eins nach B. Wir haben bei A und B jeweils einen Messapparat und messen mal die Pfeilrichtung in vertikaler Richtung (und finden, dass eins der Teilchen im Zustand oben ist, das andere im Zustand unten), mal messen wir in horizontaler Richtung (und finden als Pfeilrichtungen rechts/links)

Klingt erstmal harmlos und wenig verwirrend? Beide Teilchen sind immer entgegengesetzt, wo ist da das Problem?  Wir können uns ja z.B. vorstellen, die Teilchen werden eben mal im Zustand (oben,unten) ausgesandt, mal im Zustand (links, rechts).

Aber wenn ihr nochmal an Regel 2 denkt, merkt ihr, dass das nicht funktioniert. Nehmen wir an, die Teilchen sind im Zustand (oben,unten), aber ich messe ihren Zustand entlang der Richtung links/rechts, also horizontal. Das eine Teilchen ist jetzt also im Zustand oben und nach Regel 2 hat es eine 50%-Chance bei der Messung links/rechts für jeden der beiden Werte. Was dabei herauskommt, ist zufällig. Dasselbe gilt für das andere Teilchen.

Es müsste also prinzipiell möglich sein, dass das beide Teilchen, wenn sie im Zustand (oben,unten) erzeugt werden und wir die Pfeilrichtung horizontal messen, im selben Zustand (links oder rechts) gemessen werden, aber genau das passiert nie. Es kann also nicht sein, dass die Teilchen schon am Anfang mit einer bestimmten Richtung ausgesandt werden und sich danach nicht mehr beeinflussen, denn dann würde man bei der jeweils anderen Messrichtung eine zufällige Verteilung erwarten. (Warnung: Am Ende des Artikels erkläre ich, dass es doch eine Möglichkeit gibt, das zu tricksen….)

So weit so gut, und das weiß man spätestens seit 1935. als Einstein, Podolski und Rosen genau so eine Anordnung ersonnen haben, um deutlich zu machen, wie seltsam die Quantenmechanik ist. (Bekannt ist das ganze deshalb auch als EPR-Paradoxon.) Wer es ausführlicher wissen will, kann bei den Artikelserien klicken, da gibt es eine dreiteilige Serie “Quantenmechanik verstehen”, wo ich deutlich tiefer einsteige.)

Die beiden Teilchen sind also in einem Zustand, der sicherstellt, dass man immer entgegengesetzte Messwerte bekommt, egal wie man die Pfeilrichtung misst, rechts/links oder oben/unten, je nachdem. Vor der Messung kennen wir den Zustand aber nicht und er kann auch nicht festgelegt sein, denn wir können die Richtung, die wir messen, ja auch erst auswählen, nachdem die beiden Teilchen längst losgeflogen sind. (Trickreiche Experimente dazu erkläre ich auch hier und hier.)

Wenn man also den Zustand eines Teilchens misst (beispielsweise oben), dann weiß man jetzt, dass das andere im Zustand unten sein muss. (Misst man erst bei A in der vertikalen Richtung und schickt dann das Teilchen bei B durch eine horizontale Messung, dann bekommt man 50% Wahrscheinlichkeit für rechts und links. Auch das zeigt, dass es jetzt im Zustand oben oder unten war. [Um das nachzuweisen, muss man natürlich sehr viele Experimente machen.])

Verborgene Variablen

Aber vielleicht ist unser Ansatz ja auch zu einfach. Könnte es nicht sein, dass jedes Teilchen einfach mehr Informationen trägt als eine einzige Richtung? Es könnte doch beispielsweise eins die Information oben/rechts haben und das andere die Information unten/links? Dann würde ich immer auf beiden Seiten entgegengesetzte Ergebnisse bekommen und alles wäre konsistent? Der Zustand eines Teilchens wäre eben nur komplizierter als wir vorher dachten, jedes Teilchen hätte sozusagen eine Liste, die sagt, was es bei allen möglichen Messungen tun soll. Eine Theorie, die solche komplexeren Beschreibungen enthält, nennt man eine Theorie mit verborgenen Variablen, weil man ja nie die ganze Beschreibung herausbekommen könnte, immer nur einen Wert, denn nach der Messung wissen wir ja die Richtung eindeutig. Haben wir die Richtung oben gemessen, haben wir keine Möglichkeit mehr, jetzt noch herauszufinden, was für eine horizontale Messung auf der “Liste” des Teilchens stand, diese Information ist verloren. (Würde sie erhalten bleiben, könnten wir das mit einem cleveren Experiment nachweisen.)

Auch solche “verborgenen Variablen” lösen das Problem aber nicht ohne Weiteres. Hier kommt jetzt die viel zitierte Bellsche Ungleichung ins Spiel. (Bell hat leider nie den Nobelpreis bekommen, er war möglicherweise für den Preis 1990 vorgesehen, starb aber kurz vor der Bekanntgabe der Nobelpreisträger.) Hierzu betrachtet man, was passiert, wenn man auf beiden Seiten die Messrichtungen unterschiedlich wählt, und zwar nicht nur horizontal/vertikal, sondern mit beliebigen Orientierungen. Man bekommt dann natürlich nicht mehr genau entgegengesetzte Ergebnisse, aber wenn ich zum Beispiel auf der einen Seite vertikal messe und auf der anderen unter 45 Grad zur Vertikalen (beispielsweise Nordost/Südwest), dann beobachte ich, dass ein Wert von oben (Nord) bei A häufiger mit Südwest bei B gefunden wird als mit Nordost. Durch geschickte Wahl der Richtungen kann man jetzt statistisch zeigen, dass eine am Anfang festgelegte Liste für die Richtungen eine bestimmte Ungleichung für die Korrelationen erfüllen muss, eben die Bellsche Ungleichung. Ist die verletzt, dann weiß man, dass die Erklärung “die Teilchen bekommen am Anfang eine Liste mit den Ergebnissen für alle Richtungen und beeinflussen sich gar nicht” nicht funktioniert, dass die Teilchen also wirklich verschränkt sind. (Im Detail erkläre ich das nicht. Vielleicht finde ich ja irgendwann einen Verlag, der mein Romansachbuch zur QM veröffentlicht, dann könnt ihr es da genauer nachlesen.)

Und genau solche Experimente haben Clauser und Aspect gemacht und gezeigt, dass die Bellsche Ungleichung verletzt ist und dass es die quantenmechanische Verschränkung wirklich gibt und sie nicht einfach mit verborgenen Variablen zu erklären ist. (Anekdote am Rande: In einem Philosophie-Seminar hat mir seinerzeit ein Dozent erzählt, die Physiker würden ja alle verschwiegen, dass die Quantenmechanik längst widerlegt sei, weil ja die Bellsche Ungleichung verletzt sei. Da war ich noch klein und konnte nicht viel dazu sagen, geglaubt hatte ich es allerdings nicht. [Nein, google gab es damals noch nicht, sonst hätte sich das ja schnell erledigt.] Qualitätssicherung in der Lehre, sooo wichtig…)

Zeilinger hat dagegen verschränkte Photonen genutzt, um damit abgefahrene Effekte auszunutzen. Eins davon erkläre ich im Detail hier, aber dazu schreibe ich heute nicht mehr.

Und was bedeutet das?

Fassen wir nochmal zusammen: Wir haben zwei Teilchen, deren Pfeile immer in entgegengesetzte Richtungen zeigen, egal welche Richtung wir messen (solange die Richtung bei beiden Messungen dieselbe ist). Wegen Regel 1 und 2 können die Teilchen damit nicht einfach mit einem bestimmten Zustand losgeschickt werden. Die Verletzung der Bellschen Ungleichung zeigt, dass auch eine kompliziertere Zustandsbeschreibung nicht funktioniert.

Was sagt uns das über die Welt?

Die Standard-Interpretation der QM sagt, dass der Zustand der beiden Teilchen eben verschränkt ist. Die Pfeilrichtung ist unbestimmt, bis wir sie bei einem Teilchen messen, dann liegt sie auch beim anderen Teilchen fest (und ist entgegengesetzt). Das gilt auch, wenn beide Teilchen sehr weit entfernt sind, und man kann die Messungen genau gleichzeitig machen, so dass auch ein lichtschnelles Signal nicht von einem zum anderen käme. Das ist das, was Einstein mit “spukhafter Fernwirkung” meinte. Vornehm nennt man diese Art der Deutung des Experiments “nicht-lokal”. (Signale kann man damit aber nicht übertragen – um die Verschränkung zu sehen, muss man ja vergleichen, was auf beiden Seiten gemessen wurde, und das ist dann wieder eine gewöhnliche Informationsübertragung. Außer natürlich als Plot-Device in Science-Fiction-Stories [unschuldig-pfeif]…)

Alternativ gibt es die “nicht-realistische” Interpretation. Danach sagt unsere Beschreibung der Teilchen überhaupt nichts über die “wirkliche Welt” aus, sondern nur über unsere Kenntnis des Zustands. Misst man, ändert sich unser Wissen über die Welt, aber die Dinge, die wir zur Beschreibung unseres Wissens und der Messungen nutzen (wie die Pfeilrichtungen) haben keine echte Entsprechung in der “Wirklichkeit”, was immer das sein soll.

Die meisten Leute (ich bis vor kurzem auch) sehen nur diese beiden Möglichkeiten, entweder ist die Welt nicht-lokal oder nicht-realistisch. Es gibt aber noch ein anderes Schlupfloch: Den Superdeterminismus. Schaut nochmal auf die Beschreibung des Experiments oben: Ich habe gesagt, dass die Teilchen nicht im Zustand (oben,unten) ausgesandt werden können, weil es dann Probleme gibt, wenn ich die Messung in horizontaler Richtung mache. Aber was ist, wenn ich das nicht kann? Was wäre, wenn die Richtung, in der ich messe, und die Richtung, in der die Teilchen ausgesandt werden, zwangsläufig immer in irgendeiner Weise korreliert sind? Dann ergibt sich nie ein Widerspruch.

Klingt absurd? Ich rede hier (aus gutem Grund, die gibt es eh nicht) gar nicht über Willensfreiheit. Aber es gibt ja Experimente wie dieses hier bei denen man Sternenlicht verwendet, um die Richtung der Messung zu steuern. Irgendwie müsste also in dieser Deutung der Zustand des Sternenlichts mit dem Zustand der ausgesandten Teilchen geschickt korreliert sein, und zwar genau passend dazu, wie ich das Sternenlicht verwende, um die Apparate zu steuern. Das klingt natürlich weit hergeholt. Auf der anderen Seite löst der Superdeterminismus dafür aber das gesamte Problem von Verschränkung, überlichtschneller Festlegung des Zustands eines Teilchens durch die Messung am anderen, die ja auch nicht gerade un-absurd sind. (Mehr über Superdeterminismus erfahrt ihr auf Sabine Hossenfelders backreaction-Blog, ich hoffe, ich habe es halbwegs richtig erklärt.) Eine ausgearbeitete Theorie, wie dieser Superdeterminismus funktioniert, gibt es bisher nicht, aber die Idee ist auf jeden Fall interessant und wert, weiter verfolgt zu werden. (Und wenn man annimmt, dass das Universum sowieso am besten als Blockuniversum zu beschreiben ist, auch gar nicht so weit hergeholt – das Blockuniversum ist für den Superdeterminismus keine Voraussetzung, aber passt schon sehr gut dazu.)

Fazit

Die Quantenverschränkung ist schon ziemlich merkwürdig. Sie sagt uns, dass wir die Eigenschaften zweier Teilchen nicht getrennt beschreiben können, egal wie weit sie voneinander weg sind, und scheint zu zeigen, dass die Messung bei einem Teilchen das andere beeinflussen kann (zumindest in der Standard-Interpretation). Egal welcher Interpretation ihr anhängt, die Quantenmechanik zeigt, dass die Realität schon ziemlich seltsam ist. (Mehr dazu in diesem Artikel, den ich zu einer Zeit geschrieben habe, als ich den Superdeterminismus noch nicht kannte.) Das wird übrigens gern in der Esoterik-Szene genutzt, um allen möglichen Blödsinn zu behaupten – warum das nicht funktioniert und was diese viel zitierten Phrasen tatsächlich bedeuten, habe ich auch mal erklärt. Ein Nobelpreis für die, die diese Seltsamkeiten nachgewiesen haben und erforschen, ist auf jeden Fall hoch verdient.

Kommentare (119)

  1. #1 Christian
    Wien
    8. Oktober 2022

    Servus Martin,
    ich versuche gerade, verborgene Variablen in der Art zu finden, dass das Quant auch noch Bewegungen beziehungsweise Schwingungen in räumlichen Richtungen macht, die uns nicht zugänglich sind. Dazu stelle ich mir eine zweidimensionale Welt vor, wo zwei sozusagen verschränkte Quanten in entgegengesetzter Richtung vom Ursprung weg einer Sinuslinie folgen. Das eine Quant schwingt hin- und her, während das andere gleichzeitig her- und hinschwingt. Würden die Quanten beispielsweise tatsächlich aber einer Schraubenlinie folgen, dann tauchen die Quanten für die zweidimensionalen Beobachter abwechselnd bei +1 und -1 auf. Aber denoch in einer vorhersehbaren Abfolge: Quant A bei +1, -1, +1, -1 usw. Und Quant B gleichzeitig bei -1, +1, -1, +1 usw.
    Wenn es sich aber gar nicht um zwei Quanten handelt, die erzeugt wurden und die der Schraubenlinie folgen, sondern um viele, die sich noch dazu mit unterschiedlicher Geschwindigkeit, aber immer paarweise spiegelsymetrisch entlang dieser Schraubenlinie bewegen (ähnlich wie bei einer Explosion, wo viele Splitter mit unterschiedlicher Geschwindigkeit vom Explosionszentrum wegfliegen), dann kann das Erscheinungsmuster bei +1 und -1 für die zweidimensionalen Beobachter schon einigermaßen chaotisch aussehen. Und wenn das ganze auch noch in mehr als einer zusätzlichen Raumrichtung (und womöglich sogar noch in zusätzlichen Zeitrichtungen) erfolgt, ist es für die zweidimensionalen Beobachter dann überhaupt noch möglich, dieses scheinbar zugällige Erscheinen von einander nicht unterscheidbaren Quanten bei +1 und -1 von einem tatsächlichen zufälligen Erscheinen von bloß zwei Quanten zu unterscheiden?

  2. #2 Joseph Kuhn
    8. Oktober 2022

    Wenn das alles ganz einfach wäre, hätte es keinen Nobelpreis dafür gegeben.

    Schön, mal wieder was von dir zu lesen, auch wenn es ziemlich “unverständlich” ist. 😉

  3. #3 lioninoil
    8. Oktober 2022

    Hallo Herr Bäker,
    Bei dem schwierigen Thema bleibe ich erst mal bei der Regel 1. Zu ersten Male habe ich verstanden, was mit Überlagerung gemeint ist. Sehr gut !

  4. #4 MartinB
    9. Oktober 2022

    @Christian
    Das wären immer noch lokale verborgene Variablen. Das coole an der Bellschen Ungleichung ist, dass man damit eben zeigen kann, dass so etwas prinzipiell unmöglich ist, ohne dass man sich jedes denkbare Konstrukt angucken muss. Egal wie die lokalen verborgenen Variablen funktionieren, sie müssten ja irgendwelche Werte für die Messungen auf jeder Seite unabhängig voneinander festlegen. Und man kann zeigen, dass die Wahrscheinlichkeiten, die man in der QM bekommt, mit einem solchen Konstrukt nicht vereinbar sind.

    @Joseph
    “Unverständlich” im Sinne von “kontraintuitiv” oder “dem gesunden Menschenverstand widersprechend” ist die Sache nun mal, ich hoffe aber, dass der Artikel deutlich macht, was genau das Seltsame an der Verschränkung ist.

    @lioninoil
    Zur Überlagerung habe ich ja schon mal einen eigenen Artikel geschrieben
    https://scienceblogs.de/hier-wohnen-drachen/2020/08/11/ueberlagerungen-in-der-quantenmechanik/
    Kannst du sagen, wo es nach Regel 1 hakt, wenn du Überlagerungen verstanden hast?

  5. #5 fauv
    9. Oktober 2022

    Wenn man oben und unten vertauscht, dann wird aus rechts links und aus links wird rechts.
    Wir messen also zuerst oben und dann rechts.
    Messen wir aber zuerst rechts, dann ist das nur eine Festlegung, weil wir ja nicht wissen, ob das von oben rechts ist oder von unten rechts ist.

  6. #6 lioninoil
    9. Oktober 2022

    Nachtrag lioninoil = fauv

  7. #7 MartinB
    9. Oktober 2022

    @Fauv/Lioninoil
    Habe nicht verstanden, was mir das sagen soll. Ich nehme natürlich ein globales Bezugssystem an, in dem die 4 Richtungen eindeutig sind (N,S,O,W) oder in einem Labor Richtung Decke, Boden, Fenster, Tür oder so.

  8. #8 lioninoil
    9. Oktober 2022

    Logisch sieht es so aus
    1. Filter ……….. 2-. Filter ……….. Ergebnis
    Licht geht durch geht nicht durch . dunkel
    Licht geht durch geht durch……… hell
    geht nicht durch geht nicht durch.. dunkel
    geht nicht durch geht durch …….. dunkel

    Nur wenn beide Filter das Licht durchlassen sehen wir nach dem zweiten Filter Licht. Die Wahrscheinlichkeit dieses Falles ist 0,25, wenn der eine Filter nur senkrecht polarisiertes Licht durchlässt und der 2. Filter nur waagrecht polarisiertes Licht durchlässt.
    Wenn also die Wahrscheinlichkeit hinter dem 2. Filter größer ist als 0,25, dann ist das Licht überlagert.

  9. #9 MartinB
    9. Oktober 2022

    @Lioninoil
    Wenn du ein Polarisiertes Licht durch zwei Polfilter schickst, von denen der erste senkrecht und der zweite waagerecht ist, dann ist die Wahrscheinlichkeit immer Null, denn hinter dem ersten Polfilter ist das Photon immer senkrecht polarisiert (sonst wäre es nicht durchgekommen).

    Achtung: In meinem Artikel habe ich mit Spins argumentiert, also Spin rauf/runter sind orthogonal, erst Spin rauf, dann Spin waagerecht hat eine Wahrscheinlichkeit von 25%, wenn der Anfangszustand identische Wahrscheinlichkeit für alle Richtungen hat oder eine entsprechende Überlagerung hat. Wenn der Anfangszustand senkrecht (z.B. oben) ist, dann kommt das Teilchen durch den ersten Filter immer durch und hat dann am zweiten eine Wahrscheinlichkeit von 50%…

  10. #10 lioninoil
    10. Oktober 2022

    Martin B
    Danke, deine Erklärung ist mir zu spät eingefallen.
    Die Logik ist ein scharfes Schwert !

  11. #11 lioninoil
    10. Oktober 2022

    Also ein zweiter Versuch, die Logik zu überprüfen.
    (und nachdem ich über Spin gegoogelt habe)

    Wir haben also bei der 1. Messung ein Oben
    Bei der 2 Messung ein Rechts.
    Die Wahrscheinlichkeit ein Rechts-Oben zu haben ist 25 %.

    Wenn bei deinem Modellversuch mit der jeweilig entgegengesetzten Richtung , (das ist die wichtigste Aussage)
    festgestellt wird, dass die Letzte Messung ein Fehler war und statt rechts ein links vorliegt, dann kippen alle Ergebnisse vorher.
    Ist das deine Erklärung für Verschränkung.?

  12. #12 lion in oil
    10. Oktober 2022

    Nachtrag zur Wahrscheinlichkeit von 25 %. Das bedeutet, dass die Wahrscheinlichkeit nicht rechts und nicht oben = 75 % ist.
    Also , nicht rechts 50 %, nicht oben = 25 % ????

  13. #13 Cabernet
    10. Oktober 2022

    Ich dachte der ganze Quantenquatsch wäre schön längst entlarvt 🙁
    Nochmal 50 Jahre? 🙁

  14. #14 MartinB
    11. Oktober 2022

    @lioninoil
    Wenn das Teilchen am Anfang in einem Zustand ist, in dem alle Richtungen gleichwahrscheinlich sind, dann ist die Wahrscheinlichkeit, durch den ersten Filter zu kommen, 50%, hinter dem zweiten dann 25%. Das ist richtig.

    “Das bedeutet, dass die Wahrscheinlichkeit nicht rechts und nicht oben = 75 % ist.
    Also , nicht rechts 50 %, nicht oben = 25 % ????”
    Achtung, um das so zu rechnen, musst du ja mehrere Möglichkeiten addieren.
    1. Das teilchen wird schon im ersten Filter aufgehalten: 50%
    2. Das Teilchen wird im ersten Filter nicht aufgehalten, wohl aber im zweiten. Das ist jetzt eine bedingte Wahrscheinlichkeit und die ist 25% wie es sein muss.

  15. #15 Bernd Nowotnick
    11. Oktober 2022

    #14

    Frage: „Wenn das Teilchen am Anfang in einem Zustand ist, in dem alle Richtungen gleichwahrscheinlich sind, dann ist die Wahrscheinlichkeit…“

    Es ist nicht möglich zwischen FM- und AM-moduliert zu unterscheiden wenn man die Anfangsbedingungen nicht kennt.

    An dieser Stelle ist die allgemeine Relativitätstheorie doch schon raus, da dies nur ohne Teilchen möglich ist, oder?

  16. #16 lioninoil
    11. Oktober 2022

    Martin,
    logisch, noch eine Frage, kann das Teilchen beide Filter passieren ?

  17. #17 lioninoil
    11. Oktober 2022

    Nachtrag
    Frage zurückgezogen , muss erst einen Kaffee trinken !

  18. #18 Karl-Heinz
    Graz
    14. Oktober 2022

    Bei der quantenphysikalischen Verschränkung sind zwei Teilchen so miteinander gekoppelt, dass der Zustandswechsel des einen automatisch den des Partners verursacht. Dies geschieht instantan und unabhängig von der Entfernung.

    Was bedeutet instantan? Übersetzen ich instantan mit mit augenblicklich oder gleichzeitig habe ich bei meiner Überlegung das Problem, dass ein bewegter Beobachter, das nicht mehr gleichzeitig wegen der räumlichen Entfernung der Teilchen sieht.
    Ich hoffe meine Frage ist verständlich. 🙂

  19. #19 MartinB
    15. Oktober 2022

    @Karl-Heinz
    Gute Frage und beliebtes Problem. “Instantan” bedeutet für die Profis wie Dich “raumartiger Abstand”. Welche Messung dann das Kollabieren auslöst, ist dann nicht klar, weil ja je nach Bezugssystem die eine oder andere früher stattfindet. (Dazu steht was in meinem QM und Realität-Artikel.) Deshalb müssen Interpretationen, bei denen die WF ein tatsächlich physikalisches Objekt ist, das wirklich kollabiert (z.B: Bohm), ein bevorzugtes Bezugssystem annehmen (“preferred foliation” genannt). Das macht solche Theorien natürlich unattraktiv…
    Ein weiterer Vorteil des Superdeterminismus ist, dass man da dieses Problem nicht hat, wenn ich es richtig verstehe.

  20. #20 Karl-Heinz
    Graz
    16. Oktober 2022

    @Cabernet #13

    Ich finde es schade, dass du davon ausgehst, dass Quantenmechanik Quatsch ist. Ich vermute ein neugieriges Kind, dass mal in dir inne wohnte, ist schon lange tot. Ich mach dir aber keinen Vorwurf.

    Ich denke gerade nach was es mit der Separabilität, Separierbarkeit auf sich hat.

    Separierbarkeit = die Abtrennbarkeit von Teilen eines physikalischen Systems zu deren individueller Behandlung.

    Sie wurde in der klassischen Physik als Grundvoraussetzung der rationalen Naturbehandlung betrachtet, da zumindest eine relative Isolierung vom Rest des Universums für die Durchführung von Experimenten notwendig ist. Sowohl die nichtlineare Dynamik (kleine Variationen der Anfangsbedingungen führen zu drastisch anderen Endzuständen) als auch die Quantenmechanik (EPR-Paradoxon, Nichtlokalität) brechen allerdings mit diesem Prinzip und zeigen Nichtseparierbarkeit.

    https://de.m.wikipedia.org/wiki/Separabilit%C3%A4t_(Quantenmechanik)

  21. #21 MartinB
    16. Oktober 2022

    @Karl-Heinz
    Danke, den Begriff habe ich irgendwie bisher nicht in meinem aktiven Wortschatz gehabt, obwohl ich ja schon genau dazu geschrieben habe:
    https://scienceblogs.de/hier-wohnen-drachen/2017/08/06/die-unglaubliche-komplexitaet-des-quantenuniversums/

    Muss ich mir mal dringend merken, das Wort.

  22. #22 Karl-Heinz
    Graz
    16. Oktober 2022

    @MartinB

    Danke für den Link. Schreibst du irgendwann auch ein Buch über Quantenmechanik im Stil von Isaac oder Die Endeckung der Raumzeit?

  23. #23 MartinB
    16. Oktober 2022

    Ich habe mein Buch über QM fast fertig, aber es ist etwas/deutlich anders als Isaac – ich habe alles in eine SF-Romanhandlung verpackt, wo zwei Aliens die QM entdecken, während nebenbei die Welt unterzugehen droht…
    Was aktuell fehlt, ist ein Verlag, mal sehen.

  24. #24 Maxx
    19. Oktober 2022

    @MartinB: Danke für den Text.
    Du hast aber einen kleinen beliebten Fehler eingebaut.
    Die für meinen Geschmack etwas zu oft zittierte “spukhafte Fernwirkung” meint den Kollaps der Wellenfunktion, nicht die Verschränkung.
    Sabine hat das hier schön aufgearbeitet:

  25. #25 MartinB
    20. Oktober 2022

    @Maxx
    Ich finde das schon recht spitzfindig. Das, was bei der Verschränkung passiert, ist ja ein Kollaps der Wf, wenn ich auf einer Seite messe. MaW:
    Nicht jede spukhafte Fernwirkung ist Verschränkung, aber das, was passiert, wenn ich bei verschränkten Teilchen eine Messung mache, ist immer auch ein Kollaps und damit eine “spukhafte Fernwirkung”.
    (Es sei denn natürlich im Superdeterminismus, aber den habe ich ja explizit gesondert erklärt….)

  26. #26 Carsten
    Schwanewede
    25. Oktober 2022

    Martin hat die Bell´sche Ungleichung in dem Artikel ja schon erklärt, aber ich glaube, den Zusammenhang zwischen quantenmechanischen Interpretationen und der Bell´schen Ungleichung sollte man noch einmal klar machen. Dann ist auch der „Superdeterminismus“ nicht mehr so nebulös, sondern lässt sich besser einordnen.
    Es fing ja damit an, dass die Physiker den „Zufall“, wie er aber durch die nicht als universelles Prinzip anerkennen wollten. „Der Alte würfelt nicht“ ist eine Form des Zitats von Einstein. Die Born´sche Regel (oder Wahrscheinlichkeitsinterpretation) führt aber genau diesen Zufall in die Quantenmechanik ein. Das Ergebnis ist zufällig und folgt nur einer berechenbaren Wahrscheinlichkeitsverteilung. Der Mechanismus, der zu einem Kollaps der Wellenfunktion führt, bleibt dabei unbekannt – es ist „irgendwie“ der Messprozess in der Messkette zwischen Quanteninteraktion und Beobachter.
    Wenn ich den Zufall als Basis unserer Welt negiere, weil ich nur ein deterministisches System akzeptieren möchte, dann muss es also „verborgene Parameter“ in der Systembeschreibung geben, die ich nur nicht kenne, die aber dazu führen, dass das Messergebnis schon vor der Messung feststeht. Das ganz große Verdienst von John Steward Bell war im Jahre 1964, dass er eine Ungleichung aufstellen konnte, die es ermöglicht zu entscheiden, ob es diese verborgenen Parameter geben kann oder nicht. Das hat Martin erklärt. Und Aspect et. al. haben schlaue Experimente erdacht und durchgeführt, um am Ende festzustellen: die quantenmechanische Wahrscheinlichkeitsinterpretation stimmt, es gibt keine verborgenen Variablen. Das Messergebnis „entsteht“ im Moment der Messung. Fall erledigt.
    Es sein denn, man schaut sich die Voraussetzungen mal genauer an, die in die Bell´sche Ungleichung etwas unbewusst mit einfließen.
    Die Ungleichung ist die Folge einer klassischen Betrachtung unserer Umwelt. Sie entsteht unter der Annahme, dass Theorien lokal sind, also eine Ursache hier nicht instantan eine Wirkung an anderer Stelle hat. Die Quantenmechanik ist nichtlokal, von daher muss sie die Bell´sche Ungleichung nicht erfüllen und tut das auch nicht. Auch die Bohm´sche Interpretation der Quantenmechanik ist nichtlokal (aber enthält verborgene Parameter). Viele-Welten-Theorien umgehen das Messproblem dadurch, dass sie einfach alle Möglichkeiten als in Paralleluniversen umgesetzt betrachten.
    Auch der Superdeterminismus nutzt eine solche Lücke, um tatsächlich eine lokale Theorie mit verborgenen Variablen zu konstruieren. Dafür muss man allerdings annehmen, dass die Statistik nicht zufällig ist: der Messaufbau (bei den Photonenexperimenten die „zufälligen“ Polarisationsrichtungen) ist nicht unabhängig, sondern bereits im Vorhinein festgelegt. Der Experimentator hat in diesem Sinne keinen „freien Willen“. Was er auch tut, bereits mit dem Urknall wurde „superdeterministisch“ (eigentlich ist es einfach „deterministisch“) festgelegt, wie dieser Messaufbau einmal aussehen wird. Sabine Hossenfelder hat sich viel damit beschäftigt, und insbesondere dieser Artikel: https://arxiv.org/abs/2010.01327 zeigt, dass sich zwangsläufig (ohne finetuning) aus allgemeinen Annahmen eine Statistik in Übereinstimmung mit quantenmechanischen Vorhersagen ergibt. Es ist ja nicht unmittelbar einsichtig, dass ein Superdeterminismus ausgerechnet mit der Born´schen Regel übereinstimmen sollte. Der Superdeterminismus dieser Art müsste/könnte aber bei sehr speziellen Messproblemen Abweichungen zur Quantenmechanik zeigen – schauen wir mal. Trotzdem bleibt dieser Ansatz eine logische Spielerei. Der Preis zum Erhalt einer deterministischen und lokalen Theorie ist hoch.
    Ich hoffe, das ist eine sinnvolle Ergänzung zu Martins Artikel und hilft dem Verständnis weiter, warum man eigentlich solche Messungen an Quantensystemen macht, wie sie nun mit dem Nobelpreis ausgezeichnet wurden.
    Viele Grüße,
    Carsten

  27. #27 Carsten
    Schwanewede
    25. Oktober 2022

    Oh, da hat sich im ersten Satz des zweiten Absatzes aber ein böses Ding eingeschlichen. Das soll heißen:
    “Es fing ja damit an, dass die Physiker den „Zufall“ nicht als universelles Prinzip anerkennen wollten.”

  28. #28 MartinB
    25. Oktober 2022

    @Carsten
    Ich habe es entweder nicht richtig gelesen oder ich stimme zumindest in einem Punkt nicht überein: Die Entscheidende Frage ist nicht, ob man den Zufall negiert, sondern dass der Kollaps für eine nichtlokale Änderung des Zustands sorgt, die eben nicht durch lokale verborgene Variablen bestimmt sein kann. Auch eine lokal inhärent zufällige Theorie kann die Beobachtungen nicht erklären, insofern finde ich die Betonung des Zufalls ungeschickt. Einstein hatte zwei Kritikpunkte, das eine ist der Zufall, das andere die Fernwirkung. Dass die WF kollabiert, ist Fernwirkung, wie sie kollabiert, ist Zufall.

  29. #29 Carsten
    Schwanewede
    26. Oktober 2022

    @MartinB
    Ja, völlig richtig, das sind zwei verschiedene Punkte. Hätte ich wohl besser formulieren können.
    Die Born´sche Wahrscheinlichkeitsinterpretation (der Zufall) ist ein sehr fundamentaler Kritikpunkt Einsteins und betrifft die gesamte Interpretation der Quantenmechanik.
    Die Fernwirkung ist ein sehr spezieller Fall, der aus dem Formalismus der Quantenmechanik folgt und scheinbar zu Widersprüchen führt (EPR-Paradoxon). Hier konnte inzwischen gezeigt werden (durch die angesprochenen Experimente), dass der quantenmechanische Formalismus die richtigen Vorhersagen macht. Deshalb muss er nicht richtig sein, aber die einfache Vermutung von verborgenen Variablen innerhalb einer lokalen, realen Theorie ist keine Lösung – das führt zu Widersprüchen mit den Experimenten.
    Habe ich Deinen Punkt etwa getroffen?
    Grüße,
    Carsten

  30. #30 MartinB
    27. Oktober 2022

    @Carsten
    “scheinbar zu Widersprüchen” finde ich auch unglücklich. Da ist ja nichts in sich widersprüchlich (das fand auch Einstein nicht), es widerspricht eben nur unserer physikalischen Intuition und lässt sich *konzeptionell* schwer mit der Relativitätstheorie vereinbaren, auch wenn es da eben genau keine Widersprüche gibt. (Irgendwer schrieb mal von der “peaceful coexistence” von QM und RT).

    Der QM-Formalismus ist richtig, in demselben Sinne, wie die Newtonsche Mechanik richtig ist: Macht exzellente Vorhersagen. Ob die QM in Wahrheit in einer fundamentaleren Theorie aufgehen wird, die ganz andere Konzepte nutzt, ist natürlich eine vollkommen offene Frage. Aber ja, lokale Variablen in einer lokalen realen Theorie funktionieren nicht (bzw. nur mit anderen unintuitiven Ideen wie Superdeterminismus).

  31. #31 Aveneer
    2. November 2022

    Wusste nicht auf welchem alternativen Weg ich die Frage stellen kann…
    Werden deine Blogs mit scienceblogs.de verschwinden oder kann man auch danach noch zugreifen?
    Ich würde sie doch sehr vermissen. Ich lese immer wieder gerne auch deine alten Serien z.b zur QM.

  32. #32 MartinB
    3. November 2022

    @Aveneer
    Das weiß ich noch nicht. Angeblich sollen ja die Seiten zum Lesen weiter verfügbar sein. Ich werde aber auf jeden Fall alle meine Inhalte runterladen und ggf. nach einem Weg suchen, sie anderweitig zu veröffentlichen.
    Wie es ansonsten weitergeht, muss ich mal sehen, einige KollegInnen suchen ja gerade aktiv nach Alternativen, mal schauen.

  33. #33 Adam
    Berlin
    6. November 2022

    Wenn ich das alles richtig verstehe (was keinesfalls sicher ist), macht man mit (Super-)Determinismus aber eine riesige Pandorabüchse auf, sogar die größte aller Zeiten. Der Superdeterminismus ist ja schon fast eine Kapitulation der Physik.

    Nicht nur, dass dieser nicht falszifizierbar und damit streng genommen nicht mehr empirisch wäre – denn ich könnte ja niemals es bestätigen, noch das Gegenteil nachweisen, dass ich doch einen freien Willen hätte, dass das Universum nicht von Anfang an hat wissen können, wann ich was tun werde, wann ich welches Experiment mache, wann ich wohin schaue, sodass die Ergebnisse immer vorgegeben wären, ich dies bloß nicht wüsste, sondern Wissenschaft als solche wäre sogar anzweifelbar.

    Denn was ist Empirie eigentlich? Es ist das Erkennen von Gesetzmäßigkeiten, also Schemata, das Prognostizieren auf Basis dieser, die Untersuchung, ob die Prognose mit der Realität übereinstimmt und das einstweillige Akzeptieren oder Verwerfen dieser Schemata, solange nicht andere Ergebnisse daher kommen, die dies in Frage stellen, sodass man dann tiefer und weiter gucken muss, weil man offensichtlich noch etwas übersehen hat, das ebenfalls Einfluß hat.

    Da gäbe es dann aber nicht mehr zwingend solche Schemata, die Dinge wären “einfach nur” vorgegeben und die Ergebnisse würden bei mir die Illusion erzeugen, dass ich da solche Dinge erkannt hätte. In Wirklichkeit hätte ich nur das getan, das gesehen, was das Universum mich hätte zu jenem Zeitpunkt an jenem Ort und auf jene Art und Weise tun und sehen lassen. Die Tatsache, dass es da scheinbar ein Schema gäbe, wäre nur meine Illusion. Das einzige “Schema” wäre die Vorgegebenheit aller Dinge, Prozesse und Betrachtungen des Universums – die jederzeit von meiner Erwartungshaltung abweichen könnte, nur würde ich das nie sehen können, wenn es nicht vorgesehen war. Oder aber ich würde es sehen, weil genau dies vorgesehen war, daraus ein ganz anderes Schema ableiten oder mir den Wolf suchen, weil kein Schema passen würde (siehe die verzweifelte Suche nach Dunkler Materie und Dunkler Energie oder die Unverständlichkeit der friedlichen Koexistenz zwischen den beiden großen Theorien, ART und QM, obwohl sie in Widerspruch zueinander stehen und inkompatibel sind, klassische “Ich-bin-hier-und-nirgends-sonst”-Theorie auf der einen Seite, “Unbestimmtheits-Zufalls-Wahrscheinlichkeit gibt den Ton an-Dingens” auf der anderen, beide trotzdem sehr präzise, beide trotzdem sehr erfolgreich), weil es letztlich keins gibt, ich abermals nur etwas zu sehen bekam, was vorgegeben war. Ich betriebe also keine Wissenschaft, sondern Tautologie (“wenn es schneit, schneit es”), ohne es zu wissen.

    Ich bin Softwareentwickler. Nehmen wir mal an, ich mache da mal ein Game, wo, passend zu deinem Blog, ein Ritter gegen einen Drachen antritt, da wo Drachen wohnen. Ich möchte Geld verdienen, also soll das Game erfolgreich sein. Also muss ich ein bisschen mehr machen, als nur Ritter-Drache-Action. Ich will, dass wenn der Drache mal so richtig rumwütet, die Umgebung davon betroffen ist, also das Environment. Also soll jenes auf seine Aktionen reagieren, z.B. sollen Bäume entwurzelt werden und – physikalisch korrekt – einen Abhang runter rollen, jeder Baum dabei berechnet, wobei die Charakteristika des Baumes, der auf ihn wirkenden (Drachen-)Kraft, des Winkels, der Kraftvektoren, des Abhangs und vieler weiterer Faktoren berücksichtigt werden. Da ich kein Gott, sondern ein Programmierer bin (wobei wir uns einbilden, dass es dasselbe sei ;-)), bin ich faul und unwissend. Ich weiß also nicht, was der Spieler mit seinem Ritter tun wird, kann der KI (also dem Drachen) nur ein bestimmtes Verhaltensschema mit auf den Weg geben, weiß nicht, wann welcher Baum auf welche Weise betroffen sein wird oder ob überhaupt. Ich greife also auf eine Physik-Engine zurück, sprich eine Ansammlung von Funktionen, die genau diese Dinge für mich berechnen und sie umsetzen. Denen muss ich nur Parameter mit auf den Weg geben, z.B. wann, wo und wie der Drache einen Baum trifft usw und zwar genau in dem Moment, wo das geschieht. Das mache ich per Eventsteuerung, also formuliere ich zur Designzeit (Gottes Basteln am universellen Plan vor dem Urknall) einen allgemeingültigen Event, der immer dann ausgelöst wird, wenn der Drache etwas in seiner Umgebung trifft und übergebe dann die Daten an die Engine, lasse diese die Umgebung verändern, worauf hin dann KI und der Spieler in irgendeiner Form reagieren werden. Dann bin ich als Gott des Programmes zwar der Erschaffer, aber keinesfalls allwissend. Ich kann nicht sagen, wann der Spieler was tun wird und nur allgemein sagen, wie der Drache reagieren wird, nur allgemein sagen, was es bei der Umgebung zur Folge hat. Die Details laufen nach Regeln ab, aber das Ergebnis, also die Laufzeit, sprich das Spiel in Aktion, das Universum in seinem Lauf der Dinge, ist auch mir unbekannt, was den Vorteil hat, dass ich mein eigenes Spiel spielen kann, weil es spannend ist, weil ich eben nicht alles wissen kann.

    Was aber, wenn ich wirklich ein Gott wäre (was auch immer das ist) und alles wüsste? Dann wüsste ich welcher Spieler was, wann, wo machen wird. Ich bräuchte keine Physik-Engine, würde die Ergebnisse für jeden Spieler einfach “hard coden”, sprich fix vorgeben. Dabei könnte ich dann besonders gemein sein, denn ich würde vielleicht den Eindruck erwecken, dass da ein Schema zugrunde läge, das die Dinge Gesetzmäßigkeiten, also unpersönlichen, objektiven, ewiglich geltenden Naturgesetzen folgten. Tatsächlich täten sie nur das, was ich vorgesehen habe im jeweiligen Fall. Ich würde also die Illusion, dass solche vorlägen jedem Spieler lassen – nur dem Spieler MartinB nicht, bei dem liesse ich die Bäume einfach so in die Luft steigen. Aber auch nicht immer und für ihn nach keinem erkennbaren Schema. Ich wüsste, dass dieser Physiker ist, dass er es nicht für einen Bug halten wird, dass ihm niemand glauben wird, denn alle anderen hatten andere Ergebnisse und dass dieser nun nach Regeln suchen und nur Unintuitives, Unverständliches finden wird. Warum täte ich dies und warum gerade bei MartinB? Nun, ich bin doch ein Gott und damit niemanden Rechenschaft schuldig. Wäre dann halt so.

    Keine Ahnung, vielleicht habe ich alles auch falsch verstanden oder es war vorgesehen, dass ich dich mit einer Wall of Text nerven soll :D. Anderes Thema: wie ich schon Florian schrieb: warum probierst du nicht einfach einen YouTube-Channel aufzumachen? Da könntest du auf einen Privatblog verweisen und Werbung für dein Buch machen. Nur als Idee. Es wäre jedenfalls schade, nix mehr von dir zu lesen. Ich lese deine Sachen häufig, melde mich nur selten, weil die Gefahr groß ist, dass dann solche romanartigen Kommentare bei rum kommen 😉

  34. #34 MartinB
    6. November 2022

    @Adam
    Sorry, die Textwand hat mich etwas abgeschreckt, ich habe es nur überflogen.
    Nein, der Superdeterminismus ist keine Kapitulation. Er ist eine Hypothese, die einen auf der Suche nach einer grundlegenden Theorie leiten kann. Die Aussage ist ja nicht “Zustand ud Messung sind korreliert, d akann man nichts machen”, sondern “Zustand und Messung könnten korreliert sein, lasst uns versuchen, eine Theorie aufzustellen, die erklärt, wie das geht (und die dann über die gewöhnlich QM hinausgeht)”.

    Videos sind nicht so meins, ich mache die, wenn es notwendig ist (Corona-Lehre…), aber schreiben macht mir mehr Spaß. Bin auch selbst jemand, der lieber liest als Videos schaut.

  35. #35 Adam
    Berlin
    7. November 2022

    Nein, der Superdeterminismus ist keine Kapitulation

    Dann verstehe ich nicht Zeillinger, der bei der Wikipage dazu folgendermaßen zitiert wird:

    According to the physicist Anton Zeilinger, if superdeterminism is true, some of its implications would bring into question the value of science itself by destroying falsifiability:

    »We always implicitly assume the freedom of the experimentalist… This fundamental assumption is essential to doing science. If this were not true, then, I suggest, it would make no sense at all to ask nature questions in an experiment, since then nature could determine what our questions are, and that could guide our questions such that we arrive at a false picture of nature.«

    Das sind doch exakt die beiden Kritikpunkte, die ich auch brachte.

  36. #36 MartinB
    8. November 2022

    @Adam
    Ich denke, das ist so nicht richtig. Nimm an, es gäbe ein Blockuniversum, und keine Willensfreiheit und (evtl. bis auf Quantenzufälle) perfekten Determinismus. Trotzdem kann es Naturgesetze geben, die man durch Beobachtung herausfinden kann, genauso wie du die Regeln für’s Schachspiel herausfinden könntest, indem du nur genügend viele Partien anguckst, auch wenn du keine davon beeinflussen kannst. Klar, mit Beeinflussen ist es leichter (‘darf ich mit König im Schach rochieren?’ lässt sich am besten durch probieren ermitteln), aber das ist ja nicht zwingend. Es gibt ja auch genügend beobachtende Naturwissenschaften – Astrophysikerinnen können auch keine Schwarzen Löcher aufeinanderballern…

    Und selbst wenn – die Natur könnte so sein, und was kümmert es die Natur, dass das nicht zu unseren Ideen passt, wie die Welt oder Wissenschaft funktioniert. Dann müssen wir unser Bild der Wissenschaft eben anpassen.

    Das Ganze natürlich unter der Maßgabe, dass irgendwann jemand eine plausible Theorie des Superdeterminismus aufstellt, die auch funktioniert (und Vorhersagekraft hat).

  37. #37 Adam
    Berlin
    8. November 2022

    Trotzdem kann es Naturgesetze geben, die man durch Beobachtung herausfinden kann

    Jaein, denn wir verstehen darunter ja letztlich etwas, wie soll man sagen, “Unpersönliches” und Verlässliches. Da sehe ich auch den Unterschied zum Blockuniversum, denn dieses ist zwar auch vorgegeben, aber nicht potenziell willkürlich, wenn ich es richtig verstand.

    Wenn ein Stein runter fällt, dann tut er das nicht, weil gerade ich da hin gucke, sondern weil es ein von mir unabhängiges Gesetz gibt. Dieses Gesetz bleibt erhalten, solange sich nicht Elementares ändert, z.B. das Laufen von Kopplungskonstanten.

    Beim SD könnte ich das so nicht mehr sagen. Ich würde nur einen Effekt beobachten, der vorgegeben war. Ob dieser sich ändern wird, je nachdem wer wann guckt, könnte ich nicht mehr sagen, denn ich kenne nicht den universellen, vorgegeben “Plan”.

    Man würde also letztlich durch schiere Häufigkeit ein wiederkehrendes Schema erkennen, das zu einem Gesetz formulieren und darauf aufbauen. Genau das tut man seit jeher. Aber man tut es, weil man sich darauf verlässt, dass die Dinge Regeln gehorchen und nicht nur scheinbar, also letztlich willkürlich sind. Die Abkehr vom “Gottes Wege sind unergründlich”-Denken früherer Äras ist ja einer der Triebfedern der Empirie gewesen. Dieses archaische Denken würde mit SD Gefahr laufen eine Rennaissance zu erleben.

    Und selbst wenn – die Natur könnte so sein, und was kümmert es die Natur, dass das nicht zu unseren Ideen passt, wie die Welt oder Wissenschaft funktioniert. Dann müssen wir unser Bild der Wissenschaft eben anpassen.

    Verständlich, doch die Frage von Zeilinger, wie ich ihn verstand und auch mir wäre dann, ob das dann noch immer Wissenschaft in dem Sinne wäre, wie wir sie bis dato kannten. Ob es nicht zu sehr dazu einladen würde bei allem Unbekannten stets den SD vorzuschieben und zu sagen: “Das ist halt so vorgegeben”.

  38. #38 Adam
    Berlin
    9. November 2022

    Das hatte ich noch vergessen:

    (und Vorhersagekraft hat).

    Wie denn? Der SD liefert ja exakt das Gegenteil, die Unmöglichkeit von Prognosen durch Auflösung der Falsifizierbarkeit. Genau das ist doch der Kern des Problems.

    Wenn in deinem Beispiel mittels SD die Erklärung kommt, dass es schon zum Urknall hin festgelegt war, dass das verschränkte Up-Teilchen beim Side-Test links rum und das andere rechts rum läuft, dann ist die einzige “Erklärung” die Festgelegtheit des Urknalls selbst. In unseren Begrifflichkeiten wäre das also gleichzusetzen mit “Willkür”. Wie soll man auf Basis von Willkür Prognosen erzeugen?

  39. #39 MartinB
    9. November 2022

    @Adam
    An einer funktionierenden Theorie des Superdeterminismus wäre nichts willkürlich. Im Gegenteil, sie wäre ja komplett deterministisch – der Determinismus bezieht eben nur den Messaufbau mit ein. Insofern verstehe ich den Einwand nicht mal ansatzweise.

    “ob das dann noch immer Wissenschaft in dem Sinne wäre, wie wir sie bis dato kannten.”
    Wissenschaft wie wir sie bis dato kannten machen wir auch nicht, wenn wir inhärenten Zufall in der QM akzeptieren. Bis dahin hat man ja immer gesagt “Wissenschaft ist nur möglich, wenn identische Bedingungen zu identischen Ergebnissen führen.” Tun sie anscheinend nicht (wenn die QM inhärent zufällig ist), trotzdem geht Wissenschaft weiter.

    Es gab in der Geschichte sehr oft Ideen, was absolute Nowendigkeiten für die Wissenschaft oder gar Denknotwendigkeiten sind (z.B. absoluter Raum und Zeit, siehe Kant) – uund als wir herausgefunden hatten, dass das die Natur nicht beschreibt, haben wir das schlicht über Bord geworfen, unser Bild von Wissenschafft geändert und weitergemacht.

  40. #40 Carsten
    Schwanewede
    9. November 2022

    @ Adam, MartinB
    Ich denke nicht, dass der Superdeterminismus gegen das Prinzip der Falsifizierbarkeit verstößt. Schließlich besagt diese Theorie, dass alles deterministisch vorgegeben ist. Sofern ich also Vorhersagen mache, die auf dieser Theorie beruhen, kann ich sie prüfen und gegebenenfalls falsifizieren. Das Problem ist nur, dass ich nicht alle Parameter kenne, um in jedem Fall eine Vorhersage machen zu können. Hossenfelder (ich erwähnte es oben) meint aber, dass bei speziellen Messproblemen mit sehr wenigen dieser Parameter statistische Abweichungen zur Quantenmechanik auftreten müssten. Insofern gibt es hier sogar einen ersten Ansatz zu einer möglichen Falsifizierbarkeit.
    Wichtig ist: Im Superdeterminismus kann es keinen Zufall geben. Das bedeutet aber nicht, dass Messergebnisse nicht zufällig erscheinen können. Nehmen wir das einfache Beispiel des Münzwurfes. Je nach Anfangsbedingung des Wurfes wird die eine oder andere Seite der Münze oben liegen. Kenne ich diese Anfangsbedingung genau genug und/oder kann sie genau genug wiederholen, dann ist das Ergebnis nicht mehr zufällig, sondern ich kann gezielt Ergebnisse nach meinen Wünschen gestalten. Die exakten Anfangsbedingungen sind äquivalent zu den unbekannten Parametern im Superdeterminismus. Kenne ich sie nicht, dann erscheint das Ergebnis zufällig.
    In komplexen Systemen gibt es immer Faktoren, die nicht genau genug bekannt sind, aber das Verhalten des gesamten Systems schon bei kleinen Änderungen extrem beeinflussen. Etliche 10 Jahre Chaosforschung sollten das gut belegt haben. Ein Mensch ist physikalisch ein so komplexes System, dass wir es auch nicht ansatzweise beschreiben können. Damit ist jede Entscheidungsfindung physikalisch völlig zufällig, weshalb man korrekterweise im berechenbaren physikalischen Sinne von einem freien Willen sprechen darf.
    Der Superdeterminismus verneint das Prinzip der Nicht-Berechenbarkeit. Damit sind sämtliche Experimente und Messergebnisse vorherbestimmt, sie ergeben sich zwangsläufig aus den Anfangsbedingungen des Universums. Da ich das aber nicht nachvollziehen kann, sind die Messergebnisse für mich zufällig. Es gibt ja keinen großen „Vortäuscher“, der Spaß daran hat, mir gezielt falsche Statistiken unterzuschieben, damit ich falsche Naturgesetze postuliere. Es ist irrelevant, ob etwas nicht nachvollziehbar deterministisch oder tatsächlich zufällig ist. So lange, bis ein kluger Kopf daherkommt und eben doch, wie Bell mit den verborgenen Parametern, ein Experiment ersinnt, in dem sich die Statistiken der beiden Fälle unterscheiden. Bis das der Fall ist, können wir nur aus dem Bauch heraus sagen, was uns weniger unbequem ist: den quantenmechanischen Kollaps zu akzeptieren oder den Superdeterminismus hinzunehmen.

    Grüße,
    Carsten

  41. #41 Adam
    Berlin
    14. November 2022

    @Carsten, MartinB:

    Das heisst, dass auch die Everett’sche Viele-Welten-Interpretation mit SD hinfällig wäre, richtig? All die Möglichkeiten müssten nicht mehr in anderen Universen realisiert werden, wie die Theorie sagt, denn es gäbe diese Möglichkeiten nur gedacht. Tatsächlich wäre da aber nur eine einzige, die hiesige, denn der Zufall wäre nicht mehr länger existent. Das Teilchen müsste sich nicht länger “entscheiden”, denn die Vorgegebenheit des Urknalls hätte ihm die Entscheidung bereits vor sehr langer Zeit abgenommen, bzw. es hat nie die Freiheit einer Entscheidungsmöglichkeit gegeben. Also auch keine Notwendigkeit für beliebig viele andere Universen, deren Zahl jedesmal vergrössert wird, wenn es mehr als eine Möglichkeit gab. Weil es nie eine gab.

    Die Implikationen, die sich daraus ergäben, müssten doch sehr weit führen. Die Kopenhagener Deutung wäre betroffen, fusst sie doch auf dem Gegenteil, auf Indeterminismus. Retrokausalität beim Delayed Choice Experiment wäre nur eine Einbldung. Und auch, ob Schrödingers Katze tot oder lebendig ist, hat sich dann nicht das einzelne, radioaktive Isotop unvorherbestimmbar ausgesucht, sondern lag schon zu Beginn des Universums fest. Folglich kann sie auch nicht in einem Überlagerungszustand sein. Kann es dann überhaupt noch Überlagerungszustände geben oder wären auch diese nur die Folge daraus, dass man nicht alles weiß?

    Ihr habt beide super Erklärungen geliefert, vielen Dank dafür, aber ich bin mir nicht sicher, ob ich die Implikationen daraus wirklich verstehe, unabhängig von Präferenzen. Und warum wird der SD nur wenig verfolgt?

  42. #42 MartinB
    14. November 2022

    @Adam
    Klar hat SD weitreichende Auswirkungen – es ist nicht bloß eine andere Interpretation der QM, sondern ein Ansatz, eine grundlegendere Theorie zu finden.
    Die Viele-Welten-Theorie löst in meinen Augen gar nichts, dazu hat Sabine Hossenfelder diverse Artikel/Videos.
    Retrokausalität gibt es nicht, auch nicht in der Kopenhagener Deutung. Siehe dazu den Artikel von Ellermann (2015) Why delayed choice experiments do Not imply retrocausality
    “Und warum wird der SD nur wenig verfolgt?”
    Ich denke, das ist zum einen historischer Zufal, zum anderen läuft die Annahme, dass wir unsere Experimente nicht wählen können, massiv unserer Intuition zuwider (das kommt ja auch in dem zeilinger-Zitat zum Ausdruck). Es ist ja wirklich schwer vorstellbar, wie die Verwendung von Lichtquanten, die von weit entfernten Quellen vor Milliarden Jahren ausgesandt wurde, zum Steuern eines Versuchsaufbaus mit den Teilchen in diesem Aufbau so verknüpft sein soll, dass am Ende alles zur QM passt. Das klingt schon ziemlich seltsam/absurd. Aber auf der anderen Seite hat Sherlock Holmes ja gesagt “Wenn man alles andere ausgeschlossen hat, ist das, was übrig bleibt, die Wahrheit, auh wenn es noch so unwahrscheinlich ist.” Und es ist ja nicht so, dass nichtlokalität/Quantenzufall etc. nicht auch absurd wären, an die haben wir uns nur gewöhnt…
    Ich empfehle, einfach agnostisch zu bleiben. Wir wissen es nicht.

  43. #43 Thomas
    Berlin
    2. Dezember 2022

    Ich hab’ mal hier ein einfaches Progrämmchen geschrieben für den Alice-und Bob-Test von Weihs, Zeilinger, Aspect.
    https://www.researchgate.net/publication/364324043_Alice-And-Bob_Mathematica_file_for_entangled_photons
    Nach meiner Überzeugung ist es nicht eine Frage von Vorstellungsvermögen sondern von Logik in dem Sinne, widerspruchsfreie Sätze zu bilden. Eine Gegenseitige Abhängigkeit ohne die Übertagung von Information ist nicht möglich, dass widerspricht dem Wort “abhängig”. Korrelationen sind entweder zufällig oder abhängig. Eine ausführliche Darlegung, warum die Bellsche Gleichung falsch angewendet wird, ist auch auf deutsch zu finden bei
    https://www.researchgate.net/project/Entangled-but-without-a-spooky-action-at-a-distance
    Viele Grüße
    Thomas Kretschmer

  44. #44 Karl-Heinz
    Graz
    3. Dezember 2022

    @Thomas

    Nach meiner Überzeugung ist es nicht eine Frage von Vorstellungsvermögen sondern von Logik in dem Sinne, widerspruchsfreie Sätze zu bilden.

    Eine Gegenseitige Abhängigkeit ohne die Übertagung von Information ist nicht möglich, dass widerspricht dem Wort “abhängig”. Korrelationen sind entweder zufällig oder abhängig. Eine ausführliche Darlegung, warum die Bellsche Gleichung falsch angewendet wird, ist auch auf deutsch zu finden bei …
    Was passiert eigentlich, wenn Studenten deine Ansichten widersprechen? Müssen sie dann mit Repressalien von deiner Seite rechnen?

    Es wäre ja durchaus möglich, dass sich Studenten für Gebäudetechnik und Facility Management auch für Quantenphysik interessieren. 😉

  45. #45 Thomas
    Berlin
    3. Dezember 2022

    Nein, müssen Sie nicht. Ich schreibe hier nicht mit Titel und nicht mit Nennung meiner Hochschule. Also was soll das? Autoritätsargumente zählen nicht. Blöd ist auch das Argument, weil jemand einen Nobelpreis bekommen hat, muss es wohl richtig sein. Die Wahrscheinlichkeit ist nur subjektiv höher. Ich diskutiere das Problem bisher immer nur auf einer sehr angenehmen sachlichen Ebene.

  46. #46 MartinB
    3. Dezember 2022

    @Thomas
    In der Physik helfen Wortspiele nahezu nie weiter. Die Erfahrung zeigt, dass physikalische Begriffe oft ungünstig gewählt sind (z.B. ‘Wirkung’) oder sehr viele Bedeutungen haben (z.B. ‘Masse’). An den Begriffen und der Wortwahl rumzukritteln ist keine Physik, denn die Begriffe stehen für gut verstandene Konzepte.

    In der Physik gewinnt der, dessen mathematisch formulierte Theorie die Beobachtungen am besten beschreibt.

    “Ich finde aber, dass das Wort X nicht Y bedeuten kann” ist in der Physik wirklich nie ein gutes Argument, auch wenn es das Lieblingsargument von Relativitätstheorie-Widerlegern und sonstigen Weltformelfindern ist.

  47. #47 Karl-Heinz
    Graz
    3. Dezember 2022

    @Thomas

    Na gut man kann sich ja dein Paper mal angucken. Gibt es eigentlich schon Reaktionen auf dein Paper? Hat noch niemand auf einen eventuellen Fehler deiner Schlussfolgerung hingewiesen?

    Du musst auch die Sichtweise eines Laien, wie mich verstehen. Man muss sich erst in die Thematik einlesen. Und das dauert und dauert.
    Und man ist dann doch enttäuscht, wenn das Paper nicht das hält, was es verspricht.

    Oder ist dieses Paper gar nicht für Laien wie mich bestimmt, sondern nur für Experten?

  48. #48 Thomas
    Berlin
    3. Dezember 2022

    Martin, Wortspiele und Herumkritzeln fände ich jetzt auch nicht spannend. Im Gegenteil empfinde ich eine präzise Wortwahl als sehr hilfreich, um Phänomene zu verstehen. Daher habe ich mit Gregor Weihs einen längeren Schriftwechsel geführt, um schließlich für die Beschreibung dessen, was eine Verstellung des Modulators bei Alice und Bob bewirkt. Daher habe ich dann den Begriff Wirkung durch Einfluss ersetzt. Ich bezweifle in keiner Weise die Berechnungen der Quantenphysik sondern die Interpretation. Und da hilft es nicht: Dazu müssen mathematische Beschreibungen in Worte gefasst werden.
    Alain Aspect schreibt:”Selbst bei diesen Experimenten war es nicht möglich, Botschaften oder sinnvolle Informationen schneller als das Licht zu übermitteln, und deshalb werde ich ganz bestimmt nicht zu dem Schluß kommen, daß ein Austausch von Signalen mit Überlichtgeschwindigkeit möglich ist”. (Aspect (1983/1996):
    Gregor Weihs selbst schreibt mir: “Aus Kopenhagener Sicht hat sich also nur ihre Beschreibung des Zustandes bei Bob verändert, aber nichts Faktisches dort.”
    Insofern geht es um eine Präzisierung, was eine Information ist. Nach meiner Auffassung umfasst die minimale Information ein Bit. Und wie sollen sich die Zustände bei Alice und Bob gleichzeitig verändern, wenn sie nicht mal ein Bit austauschen? Ja, das ist ja das eigentümliche an der Quantenphysik? Ich kann mit meinen mathematischen Modellen sehr wohl aufzeigen, dass die Modulatorverstellung bei Alice einen Einfluss auf die Informationslisten bei Alice’s Detektor haben, aber nicht bei Bob und dass beide Einflüsse lokal nicht nachweisbar sind, aber durch Zusammenführung der beiden Informationen schließlich das richtige Gesetz gefunden werden kann. Ich habe gezeigt, wie das unter Beachtung von Bell und CHSH möglich ist und sowohl analytische wie numerische Ergebnisse geliefert. Ich habe schließlich gezeigt, wo bei den Versuchen von Aspect, Zeilinger und Weihs die Bellschen Gleichungen nicht richtig angewendet werden, weil die Summe transmittierter und reflektierter Photonen gleichgesetzt wird mit der Summe bei Alice oder Bob ankommender Photonen, was für einen idealen Polarisationsteiler ohne nachgeschalteten Filter ja stimmen würde. Ich habe die Versuche bezüglich der Filter zwischen Strahlteiler und Detektor überprüft und gefunden, dass diese niemals einen Transmissionsgrad von über 83 % hatten. Durch die Gleichsetzung scheint die Bellsche Ungleichung nicht erfüllt zu sein. Soweit alles klar. Ich traue der Mathematik aber ich traue auch der Logik. Mathematik ist logisch, aber ohne Sprache ziemlich wenig hilfreich.
    Ich setze mich mit den Dingen ernsthaft und gewissenhaft auseinander und bitte auf abwertende Begriffe möglichst zu verzichten. Ich freue mich aber trotzdem über deine Antwort, weil du über die Sache redest, im Gegensatz zu Karl-Heinz.

  49. #49 Karl-Heinz
    Graz
    3. Dezember 2022

    @Thomas

    Was sagt Gregor Weihs zu deinem Paper?
    Ich denke diese Frage ist sachlich.
    Ich wollte auf keinen Fall auf verletzende Weise ohne Respekt jemanden herabwürdigen.

  50. #50 MartinB
    4. Dezember 2022

    @Thomas
    “Daher habe ich dann den Begriff Wirkung durch Einfluss ersetzt.”
    Und das offenbart genau dein grundlegendes Missverständnis der Physik. Es ist (ähnlich wie bei Hilberts Programm in der Mathematik) völlig egal, ob du die Größe “Wirkung”, “Einfluss” oder “Quaddelbrumpf” nennst. Physikalische Größen sind (oft implizit) operational definiert, also dadurch, dass wir sagen “Wenn du X tust und Y misst, dann nennen wir das Z”.

    Wenn Du “Information” so definierst, dass du sagst ” Wenn Alice eine Messung macht, ändert sich bei Bob die Information, aber er kann das nicht messen”, dann ist das schlicht eine andere Definition dessen, was du Information nennst. In der Physik hat sich durchgesetzt, diese Änderung nicht “Information” zu nennen, weil wir nicht wirklich wissen, ob sich bei Bob etwas ändert – dazu gibt es nicht umsonst die endlosen Debatten über psi-epistemisch vs. psi-ontisch. Es ist ein bisschen wie in der Geschichte “Ein Tisch ist ein Tisch”, wo ein alter Mann alle Dinge umbenennt.

    Was du da glaubst ei Zeilinger et al als fehler nachgewiesen zu haben, konnte ich nicht nachvollziehen – gebe aber zu, dass ich auch nur ein paar Minuten in das exzessiv lange paper geguckt habe. Wie schon oft auf dem Blog geschrieben: Ich habe weder Zeit noch Lust, Privattheorien zu widerlegen.

  51. #51 hto
    4. Dezember 2022

    @Karl-Heinz

    Hier wirkt der “kategorische Imperativ” des nun “freiheitlichen” Wettbewerbs, da muss man es mit “Gott ist tot” halten.

  52. #52 Karl-Heinz
    Graz
    4. Dezember 2022

    @Thomas
    Für was benötig man den Toleranzwinkel und was genau soll der Toleranzwinkel sein?
    Das werden sich viele fragen, die dein Script zu lesen versuchen.

    Ach ja. Unpolarisiertes Licht und 3 Polarisationsfilter hintereinander geschalten.

    I_1= 1/2 • I_0
    I_2= 1/2 • I_0 • cos²(α_2-α_1)
    I_3= 1/2 • I_0 • cos²(α_2-α_1)•cos²(α_3-α_2)

    I_3/I_2 = cos²(α_3-α_2)

    Und du schreibst

    Durch Einfügen eines weiteren Filters C könnten wir außerdem feststellen, dass die Lichtstromminderung durch das letzte Filter (also Lichtstrom hinter Filter C zu Lichtstrom vor Filter C) völlig unabhängig ist von der Einstellung des Filters A, sondern allein abhängig ist von der Einstellung der letzten beiden Filter. In der Literatur liest man häufig, das Licht hätte seine Ursprungspolarisation vergessen, wenn es aus einem Filter austritt. Wir wissen aber ziemlich genau, dass Licht kein Gedächtnis hat, sondern formulieren lieber: Die Polarisation hinter einem Filter ist unabhängig von der Polarisation vor dem Filter. Weitere Erkenntnisse lassen sich nicht aus dem Versuch ziehen. Wir beschreiben nun ein physikalisch-geometrisches Modell.

    Also wenn ich I_3 betrachte ist dieser sehr wohl von der Einstellung von α_1 abhängig.
    Bist du da nicht ein bisschen oberflächlich und verwirrst die Leser? Das I_3/I_2 von Filter A nicht abhängig ist, sollte eigentlich trivial sein.

  53. #53 Thomas
    Berlin
    5. Dezember 2022

    Martin, deine Einwände sind doch sehr allgemein. Du schreibst ja selbst mit Wörtern aber deine Kommentare lesen sich so, als sollte man sich auf Formeln beschränken. Natürlich ist Gregor Weihs nicht mit meiner Sichtweise einverstanden. Der Begriff Privattheorien ist wenig hilfreich. Ich habe bisher noch nicht gewagt, das als Theorie zu bezeichnen. Vielleicht wird es mal eine. Das Begriffspaar Ursache und Wirkung ist sehr gut eingeführt. Ich verwende Wirkung nur in diesem Sinne. Die Vorgehensweise, den Zusammenhang zwischen Ursache und Wirkung festzustellen, ist wie folgt: Schalte möglichst alle Störquellen aus und variiere eine Eingangsgröße, das ist die betrachtete Ursache. Messe die Wirkung, die dich interessiert und stelle den Zusammenhang her. Bei dem Verschränkungsproblem gibt es nun ein Problem: Es gibt in diesem Sinne keinen stationären Zustand, da die Detektionen sich im engen Zeitabständen ändern und meine Informationsliste aus den Zeitpunkten registrierter Transmissionen und registrierter Reflexionen zusammensetzt. Da die Zeit aber weiterläuft und die Detektionen auch noch statistisch verteilt erfolgen, ist der Begriff Wirkung problematisch. Das ist nicht der Austausch von beliebigen Begriffen, sondern eine Präzisierung. Der Begriff “Einfluss” akzeptiert die Veränderlichkeit und stellt nur fest, dass die Verstellung des Modulators bei Alice einen Einfluss auf die Informationsliste von Alice oder auf die Informationsliste von Alice und von Bob hat. Im ersten Fall gibt es keine Fernwirkung, im zweiten gibt es bei gleichlangen Wegen von Photonenquelle und Alice einerseits und Photonenquelle und Bob andererseits einen unendlich schnellen Einfluss der Modulatoreinstellung auf die Informationsliste von Bob. Extremer wird’s, wenn die Stecke zwischen Bob und Photonenquelle kürzer ist als zwischen Alice und der Photonenquelle. Dann müsste der Einfluss der Einstellung am Modulator bei Alice auf Bob’s Liste schon zeitlich vor der Einstellung selbst liegen. Das heißt, die Gegenwart hat einen Einfluss auf die Vergangenheit. Solche Lösungen lehne ich ab. Daher zeigt sich, dass die Einstellung am Modulator bei Alice auch nur einen Einfluss auf die Informationsliste von Alice und nicht auf Bob’s Liste haben kann. Damit entfällt jede Fernwirkung, weil jeder Teilschritt von der Modulatorverstellung bis zur Auswertung der Korrelation dann lokalrealistisch möglich ist. Und damit klären sich gerade die von dir angesprochenen unterschiedlichen Sichtweisen.
    Mit Bildern hinterlegt ist das Thema bei
    https://www.researchgate.net/publication/365114191_Still_unanswered_questions_regarding_the_Alice-and-Bob_Tests_of_Weihs_Zeilinger_Aspect
    Ich versuche ja extra, so zu schreiben, dass man Lust dazu hat, es zu lesen…

  54. #54 Thomas
    Berlin
    5. Dezember 2022

    Karl-Heinz,
    das Thema ist sicher kompliziert, aber mein Ziel ist nicht, jemanden zu verwirren. Der Teilsatz “also Lichtstrom hinter Filter C zu Lichtstrom vor Filter C)” ist ja identisch zu I3/I2 und das ist unabhängig von I1. Dass dies unabhängig ist von I1, ist nicht trivial, es kommt uns nur so vor. Nur mal ein Gegenbeispiel aus der Praxis: Wenn ich drei Farbfilter hintereinanderschalte ist das Ergebnis anders
    Der Toleranzwinkel wird benötigt, um darzustellen, dass ein Photon mit der Polarisation alfa eine Chance hat, durch einen Polarisationsfilter mit der Polarisation beta zu kommen. Damit wird die abnehmende Wahrscheinlichkeit mit wachsender Winkeldifferenz beschrieben.

  55. #55 Thomas
    Berlin
    5. Dezember 2022

    Martin, noch eine Ergänzung zur Messung und zur Information. Was ich als Informationsliste bezeichne, lässt sich aufschreiben, von mir aus auf Papier. Siehe Linkhinweis von eben. Was sich aus der isolierten Informationsliste nicht ergibt, ist, ob die Modulatoreinstellung von Alice einen Einfluss auf diese Liste hat(te). Um dies festzustellen, müssen die beiden Listen von Alice und Bob zusammengeführt werden. Erst dann kann man die Korrelation feststellen und die schöne Sin²-Kurve.

  56. #56 Karl-Heinz
    Graz
    7. Dezember 2022

    @Thomas

    Zumindest weiß ich jetzt, was es mit dem von dir definierten Toleranzwinkel auf sich hat.
    ∫ sin(2y) dy mit den Grenzen von (α-β) bis π/2 = cos²(α-β)

    Die Fläche der Funktion sin(2y) von 0 bis π/2 ist 1.
    Dürfte dann sowas wie eine Wahrscheinlichkeitsdichtefunktion sein.

  57. #57 Karl-Heinz
    Graz
    9. Dezember 2022

    NOBELPREIS-LECTURE
    „Quantenwunderland“ inklusive Venus
    https://science.orf.at/stories/3216500/

  58. #58 Thomas
    Berlin
    9. Dezember 2022

    Der Nobelpreis, den Einstein für die Entdeckung der Photonen bekommen hat, hätte er besser für seine Relativitätstheorie bekommen sollen. Die Argumente von Anton Zeilinger kenne ich im Wesentlichen und gerade darum geht es ja. Bestimmte Aspekte passen einfach nicht. https://www.researchgate.net/publication/366158091_The_Unanswered_Question_About_the_Alice-and-Bob_Tests_of_Weihs_Zeilinger_and_Aspect

  59. #59 Karl-Heinz
    Graz
    12. Dezember 2022

    @Thomas
    Ich denke nicht, dass deine Privattheorie Zeilinger widerlegen wird. Seit wann beschäftigst du dich mit diesem Thema.
    Ich werde natürlich versuchen deine Ideen und Überlegungen nachzuvollziehen und ab und zu ein Kommentar dazu zu schreiben.

  60. #60 Thomas
    Berlin
    16. Dezember 2022

    Der Begriff Privattheorie ist nicht zielführend. Ich zeige Widersprüche auf und entwickle eine Methode, diese zu vermeiden. Ich berufe mich immerhin auf Einstein und behaupte, das er bezüglich seiner Einschätzung der Fernwirkung nicht widerlegt ist. Auch bei der derzeitigen vielverbreiteten Interpretation der Fernwirkung kommt man nämlich zu dem Ergebnis, dass die Wirkung zeitlich vor der Ursache liegen müsste. Da ich das ablehne, müssen Anton Zeilinger Interpretationen falsch sein. Daher habe ich das Problem nochmals in research-gate als Frage formuliert.
    https://www.researchgate.net/post/What_happens_in_an_Alice-and-Bob-Test_with_entangled_photons_if_the_path_from_the_source_to_Alice_is_longer_than_the_path_from_the_source_to_Bob
    Es gab einen Antwortversuch, sonst Leere. Auch hier in diesem Forum gibt es keine Antwort auf die zentrale Frage, wie denn die Fernwirkung ohne Informationsübertragung funktionieren soll. Ich bin mir aber im Klaren darüber, dass es wahrscheinlich noch 20 Jahre dauern wird, dass eine andere Sichtweise überhaupt mal ernst genommen wird. Ein Hoffnungsschimmer ist https://www.researchgate.net/publication/360006212_There_Is_No_Spooky_Action_at_a_Distance_in_Quantum_Mechanics
    Es gibt noch eine Handvoll anderer, ich suche sie noch. Karl Hess hat mal ein Buch geschrieben (Einstein was right), aber das ist nicht meine Denkweise. Er freut sich aber immer, wenn ich ihm mal was von mir schicke.

  61. #61 Karl-Heinz
    Graz
    16. Dezember 2022

    @Thomas

    Es gab einen Antwortversuch, sonst Leere. Auch hier in diesem Forum gibt es keine Antwort auf die zentrale Frage, wie denn die Fernwirkung ohne Informationsübertragung funktionieren soll.

    Ok..

    Nehmen wir mal als Modell
    1) 1.000.000 Photonenpaare werden erzeugt
    2) Die Photonenpaare haben untereinander die gleiche Schwingungsebene
    3) Die Schwingungsebene des erzeugten Photons ist zwischen 0 und 360 gleich wahrscheinlich. (bei Annahme von verborgenen Variablen)
    4) Alice und Bob
    5) Filterstellungen: a,b und c sind gleich wahrscheinlich
    a = 0°
    b= 30°
    c= 60°

    6) Filter
    T … Transmission
    R … Reflexion

    Frage:
    Wie viele Photonenpaare von den 1.000.000 Photonenpaaren nehmen bei der Messung im Mittel den Zustand
    ein für
    N(aT,bT) = ?
    N(bT,cR) = ?
    N(aT,cT) = ?

    Die Berechnung ist an und für sich trivial.
    Aber was bekommst du raus?

  62. #62 Karl-Heinz
    Graz (Österreich)
    16. Dezember 2022

    @Thomas
    Hallo Thomas

    Und wenn du Beispiel #61 geschafft hast, dann würde ich gerne wissen, was die Quantenphysik für
    N(aT,bT) = ?
    N(bT,cR) = ?
    N(aT,cT) = ?
    voraussagt.

    Welches Modell, das richtige Ergebnis liefert, kann man ja durch ein Experiment prüfen.

    Wie schon angedeutet, ist diese Berechnung sehr einfach. Ich bin schon gespannt, was du rausbekommst.
    Ich hoffe, so was darf man fragen, oder?

  63. #63 Karl-Heinz
    Graz
    16. Dezember 2022

    @Thomas

    Eine ganz einfache Aufgabe.
    Unpolarisiertes Licht fällt auf ein ideales Polarisationsfilter.
    Wieviel Prozent geht durch das Polarisationsfilter?
    Natürlich kann man so etwas berechnen.

    Lieber Thomas: Wie berechnest du diesen Fall?

    Ich denke schon, dass man das Recht hat, solche Fragen an den Lehrer zu stellen.
    Was meint ihr liebe Schüler?

  64. #64 Karl-Heinz
    Graz
    17. Dezember 2022

    Ist Quantenverschränkung bewiesen?
    Dies wurde durch Experimente bestätigt, sodass die Quantenverschränkung heute als physikalisches Phänomen anerkannt ist (bis auf wenige Abweichler). John Clauser (erster Test der Bellschen Ungleichung 1972), Alain Aspect und Anton Zeilinger erhielten dafür 2022 den Nobelpreis für Physik.

  65. #65 Thomas Kretschmer
    Berlin
    17. Dezember 2022

    zu 61, 62, 63: Das Hauptdokument ist in drei Teile geteilt. der erste Teil verzichtet möglichst auf Formeln. Der zweite Teil enthält die Formeln, der dritte Teil enthält die Simulationsergebnisse. Du findest die Fragen dort alle beantwortet. Fall etwas nicht drin steht, kannst du gerne noch einmal fragen. Du solltest aber Martins Plattform nicht dafür missbrauchen, sonst ist das für die anderen ärgerlich. Ich möchte aber auch nicht jede Frage noch einmal beantworten, die ich im Hauptdokument ausführlich beschrieben habe. Jede/r weiß doch, dass 50% der unpolarisierten Strahlung durch einen Polfilter geht. Beachte aber, dass in den Alice-und-Bob-Tests ein weiterer Filter zwischen Detektor und Polarisationsteiler eingebaut ist. Der hat max 80% Durchlässigkeit.

  66. #66 Thomas Kretschmer
    Berlin
    17. Dezember 2022

    Zu Frage 64: Die Frage ist zwiespältig.
    Zunächst ist klar, dass eine Theorie nicht bewiesen wird, sondern nur widerlegt werden kann. Siehe dazu Karl Popper.
    Dann ist die Frage, was Quantenverschränkung ist zu klären. Nach meinen Untersuchungen beschränkt sich die “Verschränkung” auf gemeinsame oder korrelierende Eigenschaften. Beispiel: Polarisation des einen ist um 90 Grad verdreht gegenüber dem anderen.
    Die Kopenhagener Interpretation geht darüber hinaus und glaubt, dass sich Eigenschaften ändern, wenn oder weil sich die Eigenschaften eines verschränkten Partners ändert. Das ist nicht bewiesen. Bisher konnte man die Messungen in sog. Verschränkungsexperimenten nicht anders interpretieren, da aufgrund der Bellschen Ungleichung klar war, dass bestimmte Korrelationen nicht auf verborgene Variablen zurückzuführen sein können. Ich als Abweichler zeige auf:
    1) Die Bellsche Gleichung ist nicht richtig angewendet worden, das die Summe der transmittierten und reflektierten Photonen am Strahlteiler zunächst durch einen weiteren Filter geschickt wurden und somit die in Summe detektierten Photonen nicht gleich zu setzen ist, mit dem Nenner der Bellschen Gleichung na. Beweis: Darlegungen von mir zu den Messungen von Weihs.
    2) Ein einfaches Modell für verschränkte Photonen kann die in Verschränkungsexperimenten erzielten Ergebnisse richtig widerspiegeln. Beweis: Simulationen von mir.
    3) Die Annahme, dass sich an verschränkten, weit entfernten Photonen etwas ändert, weil sich am verschränkten Partner etwas ändert, ist nicht ohne Informationsübertragung möglich. Beweis: Logik: Das Wort “weil” verknüpft Ursache mit Wirkung, damit ein Wirkung auftreten kann, muss es etwas geben, das wirkt.
    4) Bei ungleichen Abständen zwischen Quelle und Alice einerseits und Quelle und Bob andererseits müsste bei der “allgemein anerkannten” Verschränkungsinterpretation der Einfluss einer Verstellung des Modulators von Alice bei Bob schon vor der Verstellung selbst erfolgen.
    Beweis: Betrachtung von Ursache und Wirkung mit Ursache: Verstellung des Modulators bei A; Wirkung: Änderung der Korrelation. Darlegung, dass die Korrelation sich nur verändern kann, wenn die Verstellung des Modulatorwinkels Einfluss auf mindestens eine Informationsliste hat.
    5) Die Frage, wie die scheinbare Fernwirkung funktioniert, habe ich nun auch mit der Kryptologie verknüpft. Es zeigt sich, dass die Quelle zwei Schlüssel zu Alice und Bob sendet. Durch den Filterdurchtritt wird dieser Schlüssel einerseits auf den Polarisationswinkel von Alice und andererseits auf den Polarisationswinkel von Bob angewendet. Fügen die beiden ihre verschlüsselten Daten zusammen, können sie die Sin²-Kurve finden.

    https://www.researchgate.net/publication/366385988_The_Now_Answered_Question_About_the_Alice-and-Bob_Tests_of_Weihs_Zeilinger_and_Aspect_Encryption_is_the_Solution

    Schon 1) und 2) müssten die Fachwelt aufhorchen lassen. Mindestens mit 4) ist es ein echter Gegenbeweis. Gegenbeweis hätte ich hier gerne fett geschrieben. Wie gesagt: Popper hatte schon aufgezeigt, dass sich Theorien nicht beweisen sondern nur falsifizieren lassen. Das dies fast niemand zur Kenntnis nimmt, liegt nicht am Beweis sondern daran, dass mit dieser Tatsache Welten zusammenstürzen. Zum Glück sind es nur Gedankenwelten.

  67. #67 MartinB
    18. Dezember 2022

    @Thomas
    Auch wenn ich mich heir eigentlich nicht einmischen wollte, habe ich gerade deinen letzten Kommentar überflogen und bin bei 4 hängengeblieben. Das ist nämlich falsch. Es ist in einem QM-Verschränkungsmodell schlicht nicht klar, welche Messung den Kollaps auslöst, ob die bei A oder bei B. Es gibt hier keine klare Kausalität, also auch keine Rückwärtskausalität. Das ist lange bekannt, siehe meinen Artikel QM und Realität, und ein Problem für “objective collapse”-Modelle, die deswegen ein bevorzugtes Bezugssystem annehmen müssen (denn man braucht keine ungleichen Weglängen, es reicht ein zum Experiment bewegtes Bezugssystem wegen der SRT).
    Es ist also nicht so, dass das niemand zur Kenntnis nimmt, das ist lange bekannt, wird in der Fachwelt diskutiert etc…

  68. #68 Karl-Heinz
    Graz
    18. Dezember 2022

    @Thomas
    Das gleiche was Martin angesprochen hat, habe ich mir auch gedacht.

    Wenn man die Quantenmechanik für diesen Versuchsaufbau anwendet dann müssten
    die Formeln für
    N(aT,bT) = ?
    N(bT,cR) = ?
    N(aT,cT) = ?
    symmetrisch sein, egal ob Alice oder Bob zuerst eine Messung macht.

    Weitere Frage:
    Wenn man für diesen Versuchsaufbau das Modell mit den verborgenen Variablen nimmt, dann müssten die Formeln für
    N(aT,bT) = ?
    N(bT,cR) = ?
    N(aT,cT) = ?
    etwas anders lauten.
    Worin besteht der Unterschied?

    Weitere Frage:
    Bei der Transmission kommt cos²(λ- α) vor.
    Kannst du mir mit Hilfe eines klassischen Models erklären, wie man auf diese Formel kommt?

  69. #69 Karl-Heinz
    Graz
    18. Dezember 2022

    @Thomas

    Ich habe auch bemerkt, dass deine Ideen, Annahmen und Ausführungen zum Teil sehr holprig sind. Hast du dich wirklich ausführlich mit dieser Thematik befasst, oder hast du gedacht, die liegen sicher falsch und sie zu sichten ist reine Zeitverschwendung?

    https://en.wikipedia.org/wiki/Local_hidden-variable_theory

  70. #70 Thomas
    Berlin
    18. Dezember 2022

    Martin, danke für den Hinweis. Alles was Bob und Alice anbelangt, ist symmetrisch und solange man nur Alice und Bob betrachtet, kommt man natürlich zu den beschriebenen Ergebnissen. Einen kausalen Zusammenhang kann man nicht feststellen. Alles klar. Aber wenn man die Begriffe Ursache und Wirkung richtig anwendet, sind sie nicht direkt auf die Photonenereignisse anwendbar sondern nur auf die äußere Welt: Da stehe ich und drehe am Modulator und damit habe ich etwas verursacht. Und diese Ursache bewirkt eine Veränderung der Korrelation zwischen den Messungen bei Bob und den Messungen bei Alice. Diese wird einsteingerecht erst zeitverzögert festgestellt. Die Frage ist nun sehr praktisch, wie die Korrelation festgestellt wurde. Die einzigen Eingangsdaten für die Ermittlung der Korrelation sind die von mir so genannten Informationslisten bei Bob und Alice. Diese enthalten den Zeitpunkt für jedes Transmissionsergebnis und jedes Reflexionsergebnis. Daraus bilde ich (bzw. Herr Weihs) die Korrelation. Weitere Eingangsdaten gibt es nicht. Die Korrelation muss also entstanden sein allein aus den Informationslisten von Bob und Alice. Und darum greift das Prinzip von Ursache und Wirkung. Nur der Einfluss meiner Einstellung auf die Informationslisten kann die Änderung der Korrelation bewirkt haben. Ich erhalte also die Zusatzinformation aus der Einbeziehung der Makrowelt, weil in dieser Ursache und Wirkung gültig sind. Daraus folgt: Die Liste von Bob kann nicht durch Modulator A verändert worden sein, die Liste von Alice nicht durch Änderung von Modulator B.
    Die quantenmechanische Deutung ist, man weiß es nicht. Wenn man aber behauptet, wie es Zeilinger ja nun macht, Einstein widerlegt zu haben, dann geht es ja gerade um die sog. Fernwirkung. Ich habe aber aufgezeigt, dass die Annahme einer Fernwirkung nur erforderlich ist, wenn man glaubt, die Informationsliste von B sei durch Modulator A beeinflusst worden. Und deshalb habe ich Punkt 4 geschrieben und halte ihn für richtig für alle, die sagen, das Experiment beweise ein spontane Fernwirkung.
    In meiner schon erwähnten Schrift: “The now answered question” habe ich das Problem mit der Unbestimmtheit und Symmetrie durch Bezug auf eine Verschlüsselung erläutert. Ich würde gerne mich diesbezüglich mit einem Quantenverschlüsselungsexperten abstimmen, der nicht ganz so auf die übliche Interpretation der QM fixiert ist.

  71. #71 Karl-Heinz
    Graz
    18. Dezember 2022

    @Thomas

    Die einzigen Eingangsdaten für die Ermittlung der Korrelation sind die von mir so genannten Informationslisten bei Bob und Alice. Diese enthalten den Zeitpunkt für jedes Transmissionsergebnis und jedes Reflexionsergebnis.

    Und was ist mit den jeweiligen Filterstellung bei Alice und Bob? Sind die nicht auch wichtig für die Korrelation in Abhängigkeit der Filterstellung zur Messung?

  72. #72 Karl-Heinz
    Graz
    18. Dezember 2022

    @Thomas

    Ich würde gerne mich diesbezüglich mit einem Quantenverschlüsselungsexperten abstimmen, der nicht ganz so auf die übliche Interpretation der QM fixiert ist.

    Nehmen wir an du hättest Recht.
    Wäre dann die Quntenverschlüsselung nicht knackbar? Wür was benötige ich dann die Quntenverschlüsselung und in weiterer Folge den Quantenverschlüsselungsexperten?

  73. #73 Thomas
    Berlin
    18. Dezember 2022

    Karl-Heinz,
    zu 68 und 69
    wenn du konkret auf meine Dokumente eingehst, nachdem du sie gelesen hast, werde ich antworten. Auf solche allgemeinen “Eindrücke” wie “sehr holprig” kann ich nicht eingehen und werde deine Kommentare ignorieren. Du brauchst mir auch nichts über hidden variable Theorien erzählen. Ich bin nicht davon ausgegangen, dass da sicher jemand falsch liegt, sondern bin vom Kern der Auseinandersetzung zwischen Einstein und Heisenberg ausgegangen und habe nicht holprig sondern systematisch untersucht, welche Voraussetzungen man aufgeben muss, wenn man die Spontane Fernwirkung für nicht vorhanden hält. Ich akzeptiere aber für mich keine Autoritätsbeweise, also dass etwas richtig sein muss, weil ein Nobelpreisträger das behauptet hat. Wie ich ausführlich dargelegt habe, zweifle ich nicht die Bellsche Ungleichung an, sondern die Auswertung von Ergebnissen. Nein, ich zweifle sie nicht an, sondern sage, wo etwas nicht mit der Bellschen Ungleichung übereinstimmt. Und das ist die Stelle, wo die gemessene Summe hinter einem weiteren Filter aus transmittierten und reflektierten Photonen gleich gesetzt wird mit der Anzahl der Photonen, die zu Alice geschickt wurden. Ich werde aber deine Fragen als Anlass dazu nehmen, diesen Aspekt in meinem Profil bei Researchgate nochmals aufzubereiten. Bitte warte noch eine Woche

  74. #74 Thomas
    Berlin
    18. Dezember 2022

    zu 72
    Im Gegenteil. Die unknackbare Quantenverschlüsselung wird nicht angezweifelt, sondern liefert den Grund, warum man aus den Daten von Bob nichts herauslesen kann. Im Heisenbergschen Sinne muss dann mit dem Nichtwissen weitergerechnet werden. Das ist auch konsequent. Aber es gibt immer zwei Möglichkeiten, mit denen sich auch Heisenberg auseinandersetzte:
    a) es gibt reale Zustände und wir kennen sie nicht,
    b) der Zustand ist tatsächlich nicht vorhanden.
    Auch wenn Martin Wörter für beliebig hält, liegt hier schon ein Schwäche im Begriff “bestimmt”. Er wird verwendet für den Abschluss eines Vorganges, bei dem ein Wert ermittelt wurde und er wird in der QM verwendet, um nicht vorhandene Eigenschaften zu bezeichnen. Diese Schreibweise “nicht vorhandene” ist unüblich, jeder sagt “nicht bestimmt”. Aber damit kann man nicht mehr unterscheiden, ob hier physikalisch etwas existiert oder nicht. Daher sage ich: nicht vorhanden.
    In der Kryptografie wird nun eins klar: Es gibt Informationen, die verschlüsselt sind. Damit sind sie prinzipiell nicht lesbar, ohne den Schlüssel zu kennen. Damit wird klar, dass eine Information vorhanden sein kann, auch wenn sie nicht lesbar ist. Sie ist eben verborgen. Nur mit dem Schüssel von Alice, den sie mit max. Lichtgeschwindigkeit transportieren kann, ist die Information lesbar. Wenn man sagt, eine verschlüsselte Information ist keine Information, dann ist das gleichbedeutend mit der Aussage: “Es gibt keine Verschlüsselung von Informationen”. Das will wohl keiner. Ich schriebe das so ausführlich, weil man Martin oben behauptet hat, ich hätte eine völlig andere Definition von dem, was Information ist.

  75. #75 Karl-Heinz
    Graz
    18. Dezember 2022

    @Thomas

    Oh, ich sehe gerade es ist sehr viel Input von dir gekommen.
    Als ich noch in Ausbildung war, da hat mir niemand gesagt, was für ein verrückter Kerl Nikola Tesla war. Es wäre sicher Amüsant gewesen, das zu wissen. Heutzutage spielt Autorität für mich eher eine untergeordnete Rolle.
    Sorry das meine Sprache ab und zu über das Ziel hinaus schießt.

  76. #76 Karl-Heinz
    Graz
    18. Dezember 2022

    @Thomas

    Nein, ich zweifle sie nicht an, sondern sage, wo etwas nicht mit der Bellschen Ungleichung übereinstimmt. Und das ist die Stelle, wo die gemessene Summe hinter einem weiteren Filter aus transmittierten und reflektierten Photonen gleich gesetzt wird mit der Anzahl der Photonen, die zu Alice geschickt wurden.

    Frage: Ich nehme an du sprichst von einem Langpassfilter, der vor dem Detektor eingesetzt wird.
    a) welchen Zweck dient dieser Langpassfilter?
    b) Ist dieser Filter polarisationsabhängig?

  77. #77 Karl-Heinz
    Graz
    18. Dezember 2022

    Verschränkte niederenergetische Photonen können durch die parametrische Fluoreszenz (parametric down-conversion) in nichtlinear optischen Kristallen erzeugt werden. Dabei wird aus einem Photon höherer Energie im Kristall ein verschränktes Paar von Photonen mit je halber Energie erzeugt.

    Langpassfilter:
    Kurzpassfilter und Langpassfilter sind auch als Kantenfilter bekannt. Kennzeichnend für diese Filteroption ist der kurze Übergang zwischen Pass- und Sperrbereich. Die Wellenlänge, die Absorptions- und Transmissionsbereich voneinander getrennt, bezeichnet man auch als Kante („cut“). Dabei werden bei Kurzpässen Wellenlängen transmittiert, die kürzer sind als die Cut-Wellenlänge. Längerwellige Strahlung wird geblockt. Langpässe transmittieren dagegen langwellige Strahlung und blockieren die kürzeren Wellenlängen.

    Kantenfilter werden überall dort eingesetzt, wo bestimmte Spektralbereiche optisch voneinander getrennt werden sollen. Interferenzfilter zeigen hierbei eine weitaus steilere Kante als Farbglasfilter.

  78. #78 Karl-Heinz
    Graz
    18. Dezember 2022

    @Thomas

    Nein, ich zweifle sie nicht an, sondern sage, wo etwas nicht mit der Bellschen Ungleichung übereinstimmt. Und das ist die Stelle, wo die gemessene Summe hinter einem weiteren Filter aus transmittierten und reflektierten Photonen gleich gesetzt wird mit der Anzahl der Photonen, die zu Alice geschickt wurden.

    Der Einfluß des Langpassfilter ist nur ein Faktor von 0.8 über alle Therme der Bellschen Ungleichung.
    Das bedeutet … 🙂

  79. #79 Karl-Heinz
    Graz
    18. Dezember 2022

    Ein Paper schreibt zum Langpassfilter folgendes:
    Vor der APD ist noch ein Langpassfilter integriert, der nur Licht mit Wellenlängen oberhalb von 641 nm transmittiert. Dies schneidet zwar 10 −20 % des Fluoreszenzspektrums ab, filtert jedoch die Raman-Peaks von Diamant [Wit05, KMZW00], sowie die Reste des reflektierten Anregungsstrahls heraus und verhindert so, dass Photonen, die nicht von den NV-Zentren emittiert wurden, als Fluoreszenzphotonen detektiert werden.

  80. #80 Karl-Heinz
    Graz
    20. Dezember 2022

    Weitere Stationen und Forschungsaufenthalte führten Anton Zeilinger unter anderem ans Collége de France sowie an die Oxford University. Im Fokus seiner Forschungsarbeiten stand und steht das Phänomen der quantenphysikalischen Verschränkung, die rätselhafte Verbindung zwischen zwei Teilchen, die unabhängig von ihrer Entfernung einen identischen Zustand annehmen.

  81. #81 Karl-Heinz
    Graz
    21. Dezember 2022

    @Thomas

    So würde ich die Transmission bzw. Reflexion von unpolarisierten Licht auf ein Polarisationsfilter berechnen.
    N_T = 1/(2π) • N_0 • ∫ cos²(φ-α) dφ
    N_R = 1/(2π) • N_0 • ∫ sin²(φ-α) dφ
    von φ=0 bis 2π.
    N_T = 0,5 N_0
    N_R = 0,5 N_0

  82. #82 MartinB
    21. Dezember 2022

    @Karl-Heinz
    So sehr ich deine Texte schätze – bitte keine Zitate anderer Quellen einfach ohne Quellenangabe hier reinkopieren (bezieht sich auf #80)

  83. #83 Karl-Heinz
    Graz
    21. Dezember 2022

    @MartinB

    Oh, natürlich.
    Danke für die Info.

  84. #84 Karl-Heinz
    Graz
    22. Dezember 2022

    Die Korrelation zwischen Alice und Bob bei sehr vielen Photonen.

    TT … Transmission bei Alice und Transmission bei Bob
    TR … Transmission bei Alice und Reflexion bei Bob
    RT … Reflexion bei Alice und Transmission bei Bob
    RR … Reflexion bei Alice und Reflexion bei Bob

    P(X,Y) … Wahrscheinlichkeit der Korrelation zwischen Alice und Bob
    α … Winkelstellung des Polarisationsfilters bei Alice
    β … Winkelstellung des Polarisationsfilters bei Bob

    Modell: lokaler Realismus
    P(TT) = 1/(2π) ∫cos²(φ-α)• cos²(φ-β) dφ
    von φ=0 bis 2π.
    = 1/8 (cos(2(α-β))+2)

    P(TR) = 1/(2π) ∫cos²(φ-α)• sin²(φ-β) dφ
    von φ=0 bis 2π.
    = 1/8 (2-cos(2(α-β)))

    P(RT) = 1/(2π) ∫sin²(φ-α)• cos²(φ-β) dφ
    von φ=0 bis 2π.
    = 1/8 (2-cos(2(α-β)))

    P(RR) = 1/(2π) ∫sin²(φ-α)• sin²(φ-β) dφ
    von φ=0 bis 2π.
    = 1/8 (cos(2(α-β))+2)

    P(TT) + P(TR) + P(RT) + P(RR) =1

    Ich hoffe diese aus dem Ärmel geschüttelte und nicht gerührt Formel stimmt.
    P(X,Y) kann ein Maximum von 3/8=0,375 und ein Minimum von 1/8=0,125 erreichen.

  85. #85 Karl-Heinz
    Graz
    22. Dezember 2022

    P(aT,bT) kleiner gleich P(bT,cR) + P(aT,cT)
    a= 0°
    b = 30°
    c = 60°
    Modell: lokaler Realismus
    1/8 (cos(2(a-b))+2) = 0,3125
    1/8 (2-cos(2(b-c))) = 0,1875
    1/8 (cos(2(a-c))+2) = 0,1875

    0,3125 kleiner gleich 0,1875 + 0,1875
    0,3125 kleiner gleich 0,375

  86. #86 Karl-Heinz
    Graz
    22. Dezember 2022

    @Thomas

    Wenn zwischen Polarisator und Detektor ein Langpassfilter geschalten wird, kommen 80% der Photonen durch, und 20% nicht. Da aber die Verhältnisse gleich bleiben, hat das keine Auswirkungen auf die Bellsche Ungleichung.
    Es muss einfach nur länger gemessen werden, bis eine genügend große Anzahl von Messungen vorliegen.

  87. #87 Karl-Heinz
    Graz
    23. Dezember 2022

    Test

  88. #88 Karl-Heinz
    23. Dezember 2022

    Ich hoffe dieser Text wird nicht vom Spam-Filter vernascht. 😉
    Modell: Quantenmechanik
    P(TT) = 1/(2π) ∫cos²(φ-α)• cos²(α-β) dφ
    von φ=0 bis 2π.
    = 1/2 cos²(α-β)

    P(TR) = 1/(2π) ∫cos²(φ-α)• sin²(α-β) dφ
    von φ=0 bis 2π.
    = 1/2 sin²(α-β)

    P(RT) = 1/(2π) ∫sin²(φ-α)• cos²(α+π/2-β) dφ
    von φ=0 bis 2π.
    = 1/2 sin²(α-β)

    P(RR) = 1/(2π) ∫sin²(φ-α)• sin²(α+π/2-β) dφ
    von φ=0 bis 2π.
    = 1/2 cos²(α-β)

    P(TT) + P(TR) + P(RT) + P(RR) =1

  89. #89 Karl-Heinz
    Graz
    23. Dezember 2022

    P(TT): Unterschied zwischen lokalem Realismus und Quantenmechanik

    Ich hätte mir nicht gedacht, dass es bezüglich
    P(TT) zwischen Quantenmechanik und lokalem Realismus so einen großen Unterschied gibt.
    x steht steht für α-β.

  90. #90 Karl-Heinz
    Graz
    23. Dezember 2022

    P(aT,bT) kleiner gleich P(bT,cR) + P(aT,cT)
    a= 0°
    b = 30°
    c = 60°
    Modell: Quantenmechanik
    P(aT,bT)=1/2 cos²(0°-30°) =0,375
    P(bT,cR)=1/2 sin²(30°-60°)=0,125
    P(aT,cT)=1/2 cos²(0°-60°) =0,125
    0,375 kleiner gleich 0,125+0,125
    0,375 kleiner gleich 0,25
    Bellsche Ungleichung ist verletzt.

  91. #91 Karl-Heinz
    Graz
    23. Dezember 2022

    Ich werde natürlich noch alles zusammen fassen und auch erklären, wie man auf die einzelnen Formeln kommt und warum die einzelnen Modelle (lokaler Realismus bzw. Quantenmechanik) beim Bell-Tests unterschiedliche Ergebnisse liefern.
    Man beachte: Ich sagte die Modelle liefern unterschiedliche Ergebnisse. Das hat noch nichts mit der Realität zu tun.
    Welches Modell das Experiment bestanden hat sollte eigentlich jedem klar sein.

    Ich muss auch zugeben, ich war höchst erstaunt , was Thomas da so in seinem Paper schreibt.

    Der Transmissionsgrad auf einer Seite beträgt für verschränkte Photonen nicht 50%, sondern ca.
    41%.

    Was er wohl mit der einen Seite ausdrücken will? Was Thomas wohl unter dem Befriff Transmissionsgrad versteht? Fragen über Fragen.

    Allen inklusive Thomas Frohe Weihnachten 🙂

  92. #92 Thomas
    Berlin
    23. Dezember 2022

    Karl-Heinz, du brauchst doch nicht die gesammelten Erkenntnisse der Quantenmechanik hier stückweise unterbringen. Glaube mir, ich kenne die Ergebnisse und weiß, das der Bell-Test nicht funktioniert und daher CHSH verwendet wird.
    Es gibt ja noch das Kommunikations-Schlupfloch. Und hier die Frage: Bedeutet Kommunikation nur, dass Alice mit Bob oder umgekehrt miteinander kommunizieren, oder auch, dass ein dritter zu Alice und Bob spricht? Das ist nämlich hier der Fall: Die Quelle als dritter sendet streng korrelierte Daten zu Alice und zu Bob.
    Wie versprochen habe ich das Problem mit Bell und CHSH jetzt aufbereitet.

    https://www.researchgate.net/publication/366548510_Bell-Test_and_CHSH-Test_for_a_local_Alice-und-Bob-model
    Das ist aber nur ein Entwurf. Dazu gibt es auch ein Mathematica-file. Ich bin selbst erschrocken: Mein lokales Modell erzeugt einen CHSH-Wert von 2.82 (Bell-Parameter) und damit genau den quantenmechanisch vorhergesagten Wert. Wenn das stimmt, was ich da simuliert habe, hat dies sehr weitreichende Konsequenzen, weil dann kein CHSH-Test Nichtlokalität beweisen kann.
    Das Ergebnis stimmt aber auch genau mit meiner Sichtweise überein: Die Quelle sendet den Schlüssel. Bei der Korrelationsanalyse wird der Schlüssel genutzt, um die Winkeldifferenz zu erhalten.
    ACHTUNG: Es ist wirklich erst ein Entwurf und bedarf der Prüfung.
    Frohe Weihnachten und guten Rutsch

  93. #93 Karl-Heinz
    Graz
    24. Dezember 2022

    @Thomas

    Es gibt ja noch das Kommunikations-Schlupfloch. Und hier die Frage: Bedeutet Kommunikation nur, dass Alice mit Bob oder umgekehrt miteinander kommunizieren, oder auch, dass ein dritter zu Alice und Bob spricht? Das ist nämlich hier der Fall: Die Quelle als dritter sendet streng korrelierte Daten zu Alice und zu Bob.

    Verstehe ich nicht.
    Ich dachte es wäre immer so.

    Das bedeutet, dass es Alice und Bob verboten ist, direkt miteinander über die Werte der Bits zu kommunizieren, die Charlie ihnen geschickt hat. Alice und Bob dürfen sich jedoch vor Spielbeginn für eine gemeinsame Strategie entscheiden.

    Genieße den heutigen Tag. 🙂

  94. #94 Thomas Kretschmer
    Berlin
    24. Dezember 2022

    Ja, das ist immer so. Deshalb ist es bisher auch niemanden gelungen, Nichtlokalität nachzuweisen.
    Heute habe ich analytisch nachgewiesen, dass CHSH nicht Nichtlokalität beweisen kann, da der Toleranz-Winkel-Ansatz einen Bell-Parameter von 2 Wurzel(2) ergibt.
    BEWIESEN!
    Aber lieder nur als Mathematica-Code erhältlich.
    https://www.researchgate.net/publication/366569551_CHSH_can_not_prove_Nonlocality_This_is_the_analytical_prove

  95. #95 Karl-Heinz
    Graz
    25. Dezember 2022

    @Thomas

    #92
    Von Gleichung 6 auf 7.
    Wo ist der Faktor 1/2 hinverschwunden?

  96. #96 Karl-Heinz
    Graz
    25. Dezember 2022

    @Thomas Kretschmer

    Nimm es nicht so schwer. Ist ja nur ein Entwurf.
    Also warum fehlt in Gleichung 7 gegenüber der Gleichung 6 der Faktor 1/2.
    Wie nennt man solch einen Fehler? Betriebsblindheit? 😉
    LG Karl-Heinz

  97. #97 Thomas
    Berlin
    25. Dezember 2022

    95, 96:
    Danke für den Hinweis, das kommt davon, wenn man zu schnell ist. Da war zu viel hin- und her kopiert, aber das Ergebnis ist das gleiche, nur die Zwischenzeilen waren falsch. Schön dass du es dir angesehen hast. Zumindest konntest du feststellen, dass der Transmissionsgrad beim Bell-Original oben quadratisch eingeht und unten nur linear, entgegen deiner obigen Annahme. Das ist übrigens keine Erfindung von mir, sondern hat schon Weihs so ähnlich geschrieben. Bei CHSH sieht es anders aus, da fallen die Filtertransmissionsgrade heraus und deshalb ist CHSH auch falsch. Das grundsätzliche Problem ist, dass der Filter ganz bestimmte Photonen herausfiltern könnte. Wenn Alice nd Bob jeweils einen Mülleimer zur Verfügung gestellt bekämen und nach bestimmten Spielregeln ihre Photonen einfach wegwerfen könnten, dann können sie die CHSH-Lokalitätsbedingung verletzen!
    Hier noch das update, jedoch ohne Danksagung an dich:
    https://www.researchgate.net/publication/366548510_Bell-Test_and_CHSH-Test_for_a_local_Alice-und-Bob-model

    Ich habe nebenbei noch einen Job und arbeite weiterhin an dem Thema. Du brauchst nicht gleich zu denken, dass ich mich zurückziehe, gehe mal lieber vom Wochenrhythmus aus.

  98. #98 Karl-Heinz
    Graz
    25. Dezember 2022

    Ich habe nebenbei noch einen Job und arbeite weiterhin an dem Thema. Du brauchst nicht gleich zu denken, dass ich mich zurückziehe, gehe mal lieber vom Wochenrhythmus aus.

    Ich dachte du hast Ferien.
    Spaß beiseite. Kein Problem.

    Der Versuchsaufbau hat ja zwei Kanäle, nämlich einen (+) Kanal und einen (-) Kanal.

    Für was steht n_AB (, ) in Gleichung (2)?
    Ich vermute, dass es um die Anzahl der gemeinsamen Detektionen bei Filterstellung bei Alice und Filterstellung bei Bob handelt.
    Also in etwa so was, oder?
    n_AB (, ) = n_AB (, ) [++] + n_AB (, ) [- -] + n_AB (, ) [+-] + n_AB (, ) [-+]

    Kommt Gleichung (3), also eine Abwandlung von Gleichung (2) von dir, oder hast du sie aus einem Buch?
    Ich freue mich auf deine Atwort. 🙂

  99. #99 Karl-Heinz
    Graz
    25. Dezember 2022

    Upss, Alpha und Beta wurden geschluckt.
    n_AB (alpha, beta)

  100. #100 Karl-Heinz
    Graz
    25. Dezember 2022

    @Thomas

    Gleichung (5)

    1/2 + 1/√2 = 1.20710678118655 und ist damit sicher nicht kleiner gleich 1.

    Ich denke Gleichung (3) (4) und (5) sind Abwandlungen deinerseits von (1) und (2).
    Habe ich recht?

  101. #101 Karl-Heinz
    Graz
    26. Dezember 2022

    Du schreibst.

    The equation (1) can be
    represented quantum theoretically for the Alice and Bob experiment like this:

    Der linker Term von Gleichung (1) ist kleiner gleich 1.
    Gleichung 1 ist von (Weihs, 2000)
    So weit so gut.

    Jetzt kommt Thomas und meint er müsse sich nicht mit den Grundlagen auseinandersetzen und setzt, seine Erkenntnisse in Gleichung (1), die in meinen Augen richtig und trivial ist, ein.

    Thomas beweist, uns damit, dass
    1/2 + 1/√2 = 1.20710678118655 kleiner gleich 1 ist.

    So weit so gut … eigentlich ist gar nichts gut.
    Ich hätte mich mehr von deinem Paper erwartet.

    Sorry Thomas für meine Wortwahl.

  102. #102 Karl-Heinz
    Graz
    26. Dezember 2022

    @Thomas

    Ist die Un-Gleichung (1), nicht die von Clauser und Horne, die eine schwächere
    Annahme hat als CHSH.

  103. #103 Karl-Heinz
    Graz
    27. Dezember 2022

    @Thomas

    Nur komisch, dass beim Paper “Strong Loophole-Free Test of Local Realism” die system detection efficiencies sehr wohl berücksichtigt wird. Man ist sich deren Bewusst, was du komischerweise verschweigst.
    Bist du Dir sicher, dass du dieses Paper genau gelesen hast?

  104. #104 Carsten
    Schwanewede
    30. Dezember 2022

    Moin Thomas, Karl-Heinz,
    hier hat sich ja eine mächtige Diskussion gebildet, inklusive einer Reihe von kurzen Abhandlungen von Thomas. Ich bin durch einiges durchgegangen und habe versucht, Thomas Gedankengänge nachzuvollziehen. Was mir auffällt: Ich denke, Thomas hat einige grundlegende Dinge der Quantenmechanik nicht oder vielleicht auch falsch verstanden. Daraus resultieren dann für ihn Widersprüche, die er herauszuarbeiten versucht und die für ihn den experimentellen Nachweis der Verletzung der Bellschen Ungleichungen zweifelhaft erscheinen lassen. Ich möchte versuchen, ein paar dieser Dinge zu klären:
    – die unterschiedlichen Weglängen bzw. Ursache/Wirkungsfragen (die Frage habe ich verstanden, die Lösung im zweiten Artikel nicht).
    Der Knackpunkt ist, dass es gar keine Ursache-Wirkung-Beziehung gibt. Verschränkte Photonen sind ein gemeinsames Quantenobjekt. Die Einstellung des Modulators “bewirkt” nichts. Im Moment der Messung wird die Orientierung beider Photonen festgelegt. Nur die Information darüber breitet sich mit Lichtgeschwindigkeit von der jeweiligen Messposition aus. Es kann an verschiedenen Punkten im Raum bzw. bei verschiedenen Beobachtergeschwindigkeiten die Information der einen oder anderen Messung früher zur Verfügung stehen, aber es gibt kein Ursache-Wirkung-Verhältnis. Das hat Martin schon gut erklärt, oben hat er ja auch darauf hingewiesen: https://scienceblogs.de/hier-wohnen-drachen/2012/10/10/quantenmechanik-und-realitat/5/
    Die Filtereffizienz bzw. Detektionseffizienz:
    In der Tat war das insbesondere im Beginn der Bell-Experimente ein logisches loophole. Es könnte ja sein, dass vermehrt gerade solche Photonen nicht nachgewiesen werden, die bestimmte Eigenschaften haben. Wie Karl-Heinz aber anmerkte, konnte dieses loophole inzwischen geschlossen werden. Es ist auch nicht richtig zu sagen, das bei einer geringeren Effizienz als 82,8% der Nachweis einer Verletzung der Bellschen Ungleichung nicht möglich ist. Sie ist möglich (das ist ja das experimentelle Ergebnis), lässt jedoch auch noch eine andere Deutung zu (wie merkwürdig auch immer die dann aussehen müsste).
    Insgesamt macht das auf mich den Eindruck, als ob Du Dich da etwas verzettelt hättest, Thomas. Bisher konntest Du bei mir jedenfalls keine Zweifel an der Quantenmechanik und den Experimenten wecken.
    Viele Grüße,
    Carsten

  105. #105 Karl-Heinz
    Graz
    30. Dezember 2022

    @Carsten

    Es freut mich, dass auch du dich in die Diskussion einbringst.
    Ich hatte mich schon gefragt: Wo sind sie denn, all die Intelligenten?

  106. #106 Carsten
    Schwanewede
    3. Januar 2023

    Danke, Karl-Heinz. Wobei ich allerdings denke, dass die weitaus meisten, die sich hier an Diskussionen beteiligen, über ein gutes Maß an Intelligenz verfügen. Es scheitert manchmal wohl eher daran, dass man sich in eine Sache zu sehr verbeisst. Aber ich bin darauf gespannt, wie Thomas´ Rückmeldung wohl aussieht.
    Grüße,
    Carsten

  107. #107 Karl-Heinz
    Graz
    4. Januar 2023

    @Carsten

    Aber ich bin darauf gespannt, wie Thomas´ Rückmeldung wohl aussieht.

    Ich denke Thomas wird uns nicht glauben obwohl du es sehr schön zusammengefasst hast.

    Das hintert jetzt natürlich keinem, aufzuzeigen wie falsch manch seiner Beweisführungen sind. Es geht um das Experiment von Weihs. Ein Experiment zum Testen der Bellschen Ungleichung unter Einsteinscher Lokalität

    Es gibt ja sehr viele Abarten der Bellschen Ungleichung.
    Eine davon sieht so aus
    (I) linker Term ≤ 1 Modell: lokaler Realismus
    (II) linker Term ≤ 1/2+1/√2 = 1.2… Modell: Quantenmechanik

    Wenn man von der Bellschen Ungleichung spricht meint man immer Gleichung (I). Gleichung (II) kann die Ungleichung (I) verletzen.
    Filter und Detektoren sind in der Praxis nie ideal.
    Thomas beweist jetzt, dass der Detektionseffizienz bei diesem Experiment größer gleich 0,828 sein muss, was bei diesem Experiment mit einem kleinerem Wert natürlich nicht der Fall ist.
    Witzigerweise geht er bei seinem Beweis davon aus, dass ein “system detection efficiencies” einen Einfluss auf Gleichung (II) hat. Gleichung (I) ist nach seinem Ansatz nicht davon betroffen. Nachdem er diese Offensichtlichkeit nicht erwähnt hat, nehme ich an, dass dies ihm noch gar nicht aufgefallen ist.

  108. #108 Karl-Heinz
    Graz
    4. Januar 2023

    ≤ sollte das Zeichen ≤ sein.

  109. #109 Karl-Heinz
    Graz
    4. Januar 2023

    Seit Anfang der achtziger Jahre wurden zahlreiche Experimente durchgeführt, die immer wieder zeigten, dass Einstein Unrecht hatte und die Quantenmechanik richtig ist. Doch diese Experimente ließen Einsteins Sympathisanten ein Hintertürchen offen: Es konnte nur ein Bruchteil der Teilchen registriert werden. Deshalb konnten die Gegner der Quantenmechanik sich regelmäßig alternative Theorien ausdenken, die erklärten, warum ausgerechnet der registrierte Bruchteil sich scheinbar so verhielt, als ob die Gesetze der Quantenmechanik richtig wären.

    Quelle: https://www.wissenschaft.de/technik-digitales/einsteins-spukhafte-fernwirkung-ist-realitaet/

  110. #110 Karl-Heinz
    Graz
    4. Januar 2023

    Quelle:
    https://www.spektrum.de/magazin/verschraenkung-total/827650

    Das stärkste Argument der Verfechter des klassischen Realismus war, dass bei solchen Messreihen viel mehr verschränkte Teilchenpaare produziert wurden – in den Versuchen von Alain Aspect in Paris handelte es sich um Lichtquanten –, als hinterher gemessen werden konnten. Mit anderen Worten, die Verschränkungs-Experimentatoren schlossen notgedrungen stets von einer sehr kleinen Stichprobe gemessener Werte auf die Verteilung einer sehr viel größeren Gesamtheit von nicht gemessenen Werten. Also konnte es immerhin sein, dass aus irgendeinem Grund die Bellsche Ungleichung zwar bei der Stichprobe verletzt wurde, nicht aber bei der Gesamtheit – und durch diese statistische Hintertür wäre der lokale Realismus vielleicht doch noch zu retten gewesen.

  111. #112 Karl-Heinz
    Graz
    4. Januar 2023

    @MartinB
    Danke für den Link

  112. #113 Karl-Heinz
    Graz
    8. Januar 2023

    @Thomas

    Ich beziehe mich auf die Simulation ” Ein Alice-und-Bob-Test ohne gruselige Fernwirkung”
    https://www.researchgate.net/figure/Preview_fig1_364324043

    Du schreibst: Die beiden getrennten Polarisatoren sind nicht verbunden. Das mag ja stimmen.

    Das Beste aber kommt jetzt. Für die Entscheidung, ob beide verschränkte Photonen transmittieren, verwendest du die Information beider Filterstellungen. So habe ich mir das nicht vorgestellt. Woher soll das eine verschränkte Photon für die Entscheidung (Messung) wissen, welche Filterstellung das andere Photon hat?

    Eine fernwirkungsfreie Deutung hast du damit leider mit deiner Simulation nicht bewiesen.

  113. #114 Karl-Heinz
    Graz
    8. Januar 2023

    @Thomas

    Und in Tabelle 1 von Entanglement.pdf haben sich auch etliche Fehler eingeschlichen.

    Zum Beispiel 1/(4pi) ~ 0,32
    Gemeint ist hier wahrscheinlich 1/pi

    ind das Verhältnis QM/TA oder ist hier das inverse Verhältnis gemeint?

    Und das dein TA andere Werte liefert als QM.
    Hast du dafür Beweise durch ein Experiment?

  114. #115 Sinclair ZX-81
    14. Februar 2023

    Danke für den interessanten Artikel.

    Was ich nicht verstehe:
    Könnte man nicht eine deterministische Interpretation der QM leicht durch die Annahme konstruieren, dass unser Universum eine Simulation(z.B. Zellulärer Automat a la game-of-live) ist, in der probabilistische Elemente (Kollaps der Wellenfunktion) einfach durch einen (pseudo)-Zufallsgenerator “implementiert” sind.
    Nach dem Motto: “if( random_generator() < wave_function_probability(t) ) then measured_result = a; else measured_result = b;)

    Eine solche Simulation mit eingebautem Pseudozufallsgenerator zur Implementierung des Phänomens "Kollaps der Wellenfunktion" wäre als Turing-Maschine per Definition völlig deterministisch.

    "Simulation" Mag esotherisch klingen, wäre aber für mich viel einfacher zu akzeptieren als die herkömmlichen QM-Interpretationen ("freier Wille", "many worlds", "Spukhafte fernwirkung")

  115. #116 MartinB
    14. Februar 2023

    @Sinclair
    Solange das Programm dann auch nicht-lokal auf die elemente zugreifen kann (das müsste es aber), würde das sicher gehen.
    Ist dann halt nur keine Physik mehr, dann kannst du auch gleich annehmen, dass du ein gehirn im tank bist oder in der Matrix steckst oder sonst was. Kann sein, löst aber in dieser Welt nichts.

  116. #117 MartinB
    14. Februar 2023

    PS: Siehe auch oben die Diskussion so etwa bei Kommentar #35

  117. #118 Sinclair ZX-81
    14. Februar 2023

    Wie gesagt, es geht mir nicht um Hirn-Im-Tank-Esoterik oder Philosophie, sondern um eine rationale, deterministische Interpretation der QM. Jedenfalls finde ich die Vorstellung eines Universums als Turingmaschine viel weniger esoterisch als “freier Wille” oder “spukhafte Fernwirkung” oder “unendlich viele Paralleluniversen”

  118. #119 MartinB
    15. Februar 2023

    @Sinclair
    Aber mit “Es ist nur eine Simulation” wirft man eben die Flinte ins Korn, dann kann man auch Magie oder Gott als Grund angeben. (Ist eigentlich erstaunlich, dass noch niemand drauf gekommen ist, die Quantenverschränkung als Gottesbeweis zu nutzen, aber wahrscheinlich gibt es das und ich kenne es nur nicht.)
    Aus “alles ist eine Simulation” lässt sich nun mal keine testbare wissenschaftliche Beobachtung ableiten.

    Wir leben in diesem Universum, egal ob es eine Simulation ist oder nicht, und versuchen, die Regeln innerhalb unseres Universums herauszufinden, so gut das geht.